You are on page 1of 174
NAC OSCE A Comprehensive Review NAC OSCE A Comprehensive Review First Edition ConadaPrep) Copyright @ 2011, Canadaprep. ‘All rights reserved. No pact of this publication may be reproduced or transmitted in any form o: by any means, electronic or mechanical, including photocopy, recording, or any information storage and retrieval system, without permission in writing from the publisher, Reproducing passages from this book without such written permission is an inftingement of copyright law. Care has been taken to confirm the accuracy of the information presented and to describe generally aceepted practices. Howeves, the authors, editors, and publishers are not sesponsible for errors or omissions or for any consequences from application of the information in this book and make no warranty, expressed or implied, with respect to the contents of the publication,’The authors, editors, and publishers have exerted every effort accordance with the current to ensure that drug selection and dosage set forth in this text oF recommendations and practices at the time of publication. However in view of ongoing research, changes in ‘government regulations, and the constant flow of information relating to drug therapy and drug reactions, ‘the reader is urged to check the package insert for each drug for any change in indications and dosage and for added warnings and precautions. This is particularly important when the recommended agent is a new or infrequently employed drug. This publication has not been authored, reviewed or supported by the Medical Council of Canada, nor is it endorsed by the Medical Council as a review material for the NAC OSCE. PREFACE ‘This book was written due to the lack of preparation material available for the National Assessment Collaboration (NAC) Objective Structured Clinical Examination (OSCE). As an International Medical Graduate (IMG) preparing for clinical and written exams in Canada, there is no comprehensive review textbook available for the NAC OSCE. Due to the lack of resource materials, many students are forced to study ftom sources that are not relevant to the NAC OSCE. This eventually hampers the candidate's score in the examination. ‘This book aims to guide you through the steps of the NAC OSCE and ensures that you are well prepared and a step ahead of the competition. A great effort has been put into collecting and organizing relevant content for both the clinical OSCE stations and the written therapeutic exam, ‘Written by medical graduates who are oriented to the NAC OSCE, this comprehensive review can be used as a framework, complementing your clinical skills and therapeutic knowledge as you prepare for the examination, This book is dedicated to all the IMGs preparing for the medical licensing examinations in Canada. “And most important, have the courage to follow your Beart and intuition. They somehow already know what yet truly want to Become. Everything else is secondary.” ~ Steve Jobs TABLE OF CONTENTS Introduction to NAC OSCE General Information sa. Registration for NAC OSC Fees Examination station NAC OSCE scoring Saumple of Therapeutic written test. : Sample clinical case station ...... "3 5 ‘Therapeutic Guidelines Maticine Cardioto, Dermatolo Endocrinolog - Gastroenterology Hematology .. Infectious Diseases Neurology’. Qrolarmgciogy Pulmonol Rheumatol Nephrology/Ucclogy Emergency Medicine sss Counseling (smoking/alcohoi) Obstetrics & Gynecology Sexually transmitted infections . Usinary tract infection Vulvovaginitis Pelvic inflammatory disease Dysfunctional uterine bleeding. Dysmenorthea ‘ + oe " 40 Endometriosis pe = 40 Hormone replacement therapy Emergency contraception .. Group B Streptococcus in pregnancy Pregnancy induced hypertension Ectopic pregnancy .. Hypezemesis gravidarum Drugs contraindicated in pregnancy Pediatrics ‘Acute bronchiolitis. a : Acute otitis media. : 2 : Asthma| we it ve Bacterial tracheitis Bacterial pneumonia 46 Croup (Laryngotrachcobronchitis) 148 Ebae 48 Seemed plants (Group A Saepincocas) 48 ‘Whooping cough (Pertussis) A Bacterial mieningitis Febrile seizures Urinary tract infection .. Dose of tylenol. Immunization schedule TABLE OF CONTENTS Psychiat Ustedon Mania Panic disorde Social phobia wn General anxiety disorder Obsessive compulsive disorder. Post traumatic stress disorder... Dementia _ Depression. Paychosis . ‘Mood stabiliz ‘Medications causing sexual dysfunction .. Substance abuse... Clinical Examination Abdominal... Cardiovascular Peripheral vascular » Respiratory examina Central nervous system Upper limb neurological 7 Lower limb neurological « Musculo-skeletal system : Spine/Back . HID saincussisotonmnt reat Knee Breast examination ~ ral State Examination Clinical cases Protocol for history taking... Medicine Cerebrovascular attack... Digoxin toxicity... Infectious mononucleosis (sore throat) ‘Migraine (Headache) . Myocardial Infaretion(Chest pain) Pneumonia Post exposure prophylaxis for HIV Pulmonary eubolisth Seizure disorder ‘Temporal arteritis onmren di hee stetrics and Gynecology Abortion ‘Antenatal vis Ectopie pregnancy TABLE OF CONTENTS. Infertility. OCP counseling... Pelvic inflammatory disease Placenta previa. Pre eclampsia vacsnesnn Pediatrics Failure to thrive Febrile seizure... Measles " % Neonatal jaundice . Primary nocturnal enuresis, Pylorie steno Speech delay.. Bulimia. Delirium . Dementia... Depression Mania Panic attack Schizophrenia Suicide cree Surgery Back Pain . Basal cell carcinoma Benign prostatic hyperplasi Carpal Tunnel Syndrome Deep Vein Thrombosis Diabetic foot Difficulty Hematemesi Neck swelling Pain abdomen Peripheral vascular disease .. Post operative fever .. Solitary lung nodule ‘Thyroid mass Trauma. Counseling Breast feeding... 2 2 159 Child abuse Domestie violence wn. Hormone replacement therapy. ‘Mammogram Immunizz Obesity . Smoking Introduction to NAC OSCE | General Info 1 Introduction to NAC OSCE General Information ‘The National Assessment Collaboration, or NAC OSCE, was established to provide a system that streamlines the assessment of IMG medical knowledge and clinical stills throughout Canada. Many international medical graduates (IMGs) find that the path to obtaining 1 medical license in Canada challenging and difficult to navigate. Different provinces and territories have their own system for assessing IMG medical knowledge and clinical skills. Comprised of a number of federal and provincial assessment and educational stakeholders, the NAC OSCE aims to streamline the evaluation process through which an IMG must navigate to obtain a license to practice medicine in Canada. Through such a system, an IMG's path to licensure would be the same, regardless of the jurisdiction in which he or she is being assessed. The NAC OSCE has replaced CEHPEA’ Clinical ‘Examination 1 (CE1), which was unique to Ontario, Registration for NAC OSCE Registration for the NAC OSCE in Ontario starts in Novernber, with the deadline in January the next year. Candidates are advised to complete their registration within this time-frame, Once the deadline is over, the candidate will not be able to register for the NAC OSCE for the entire year. The exams are scheduled for ‘March, June, August and September. Visit www.mee.ca and www.cchpea.ca for updated information. Fees Application Fee: $200 which is non-refundable, NAC OSCE Fee in Ontario: $1850 and Exam Date Change Fee: $100 All fees are in (CAD) Canadian Dollars, Examination station “The format for the National Assessment Collaboration (NAC) Objcctive Structured Clinical Examination (OSCE) consists of 12 stations based on presentations of clinical scenarios. For a given administration, each candidate rotates through the same series of stations. Each station is 10 minutes in length with two minutes berween stations. At each station, a brief written statement introduces a clinical problem and outlines the candidate's tasks (c.g take a history, do a physical examination, ctc.).In each station, there is atleast one standardized patient and a physician examiner. Standardized patients have been trained to consistently portray a patient problem, Candidates should interact with standardized patients as they would with their own patients. The physician examiner observes the patient encounter, For most stations, the candidate will be asked to respond to a series of standardized oral questions posed by the physician examiner after seven minutes with the standardized patient. There are no rest stations. Orientation videos hetp://www:mce.ca/en/video/QEIL-Orientation/index html 2 NAC OSCE | A Comprehensive Review ‘The examination includes a separate written test of candidates’ therapeutic knowledge. This component lasts 45 minutes and consists of 24 short-answer questions testing the candidates’ knowledge of therapeutics for patients across the age spectrum and related to pharmacotherapy, adverse effects, disease prevention and health promotion, NAC OSCE scoring ‘The candidate’s total examination score will be determined by combining the scares on the OSCE component with the scores on the therapeutics component. The OSCE score contributes 75 per cent of the total score and the therapeutics score contributes 25 per cent of the total score, For reporting purposes, the NAC total ‘examination scores are reported on a scale with a distribution ranging from 0 1 100 with a fixed passing mark of 65. Number of times candidates can take the examination Starting in 2011, the NAC OSCE can be attempted once per Canadian Resident Matching Service (CaRMS) cycle. If you pass the examination, you can register for the examination a maximum of two additional times if your eligibility is maintained. Regardless of whether you pass or fail, you can only take the examination three times. Ifyou take the examination more than once, the most recent zesult will be the only valid result, Sample of Therapeutic written test Question: An otherwise healthy 65 year old woman presents with a 3 week history of aching and morning stiffness in both shoulders with difficulty dressing. She has no temporal artery tenderness, headache, jaw pain or visual disturbance. Her ESR (Exythrocyte sedimentation rate) is 100 and you have made the diagnosis of POLYMYALGIA RHEUMATICA (PMP). ‘What would you choose as the drug of first choice for initial medical therapy? (Drug, dose, route of administration and duration are required.) Answer: —— Answer key the marker receives: PREDNISONE 7.5 20 mg PO od for 2-4 weeks following resolution of symptoms Question: An otherwise healthy 55 year old male with a history of childhood “chickenpox” presents with a 2 day history of painful unilateral vesicular eruption in a restricted dermatomal distribution. You make a diagnosis of HERPES ZOSTER (shingles). ‘What would you choose as the drug of first choice to promote healing and lessen the neuropathic pain? (Drug, dose, route of administration and duration are required.) Answer: Answer key the marker receives: VALACYCLOVIR (VALTREX ®) 1000 mg PO tid X 7 days OR FAMCICLOVIR (FAMVIR ®) S00 - 750 mg PO tid X 7 days OR ACYCLOVIR (ZORIVAX ®) 800 mg PO 5X / day X 7 days) Introduction to NAC OSCE | General Info ‘Sample Clinical Case Station Example instruction written outside the station David Thompson, 59 years old, presents to your office complaining of jaundice, Tn the next 7 minutes, obtain a focused and relevant history After the 7 minutes, you will be asked to answer questions about this patient, Example of post encounter questions (QU. The abdominal examination of David Thompson revealed no organ enlargement, no masses and no tenderness. What radiologic investigation would you first order to help discriminate the cause of the jaundice? Q2. Ifthe investigations revealed that this patient ikely had a post-hepatie obstruction, what are the two principal diagnostic considerations? QB. What madiologie procedure would you consider to elucidate the level and nature of the obstruction? Therapeutic Guidelines | Medicine Therapeutic Guidelines Medicine 1. Cardiology Acute Myocardial Infarction : Immediate management in ER 1. Beta blockers: Inj Metoprolol 2.5-5 mg rapid IV q2-5 min, upto ARTE MUTREATMENT 15 mg over 10-15 minutes, then 15 minutes after reeciving 15 mg IV. B: Beta Blockers 2, Then 50 mg PO q6h x 48 hours, then 50-100 mg PO BID. M::Morphine Sulphate 3. Inj Morphine Sulfate IV 2-5 mg every 3-30 min pn Cou A (If pain not relieved with 3 Sublingual Nitroglycerins) a pee " 4, Oxygen by nasal cannula at 4 liters per minute 5. Sublingual Nitroglycerin 0.3-0.6 mg, q5min up to 3 times. 6, Non-enteric coated Aspirin 325 mg PO. 7. Cardiology Consultation Post MI drugs Drugs Benefits Side effects Contraindication ACE Inhibitors 4 mortality Hypotension/dizziness | lateral renal artery Ramipril - 10mg hs Prevents ventricular remodelling | Hyperkalemia stenosis Lsinopril - 10mg od + proteinuria Angioedema Hof angioedema Enalapril — 20mg od Renal insufficiency Pregnancy Captopril 50mg tid ‘Cough, taste changes ARB 4 mortality Angicedema Valsartan ~ 160mg bid | proteinuria |Cough, taste changes Candesartan - 32mg od Beta Blocker + mortality Decreases BP&HR | Severe/poorly Metoprolol ~ 100mg bid 4 sudden death, reinfarction & | Dizziness fatigue controlled asthma Atenolol ~ 100mg od archythmias Sexual dysfunction _|2/ird degree heart Carvedilel - 25mg bid Cardioselective :preferredfor | May mask hypoglycemia | Dlock Propranolol — 60-80 tid mild asthma and diabetes eaentr HR<50, SEP<90 rdiogenic shock | Cocaine use Statins + mortality in post MI patients | Gl upset, muscle aches, | Active liver disease, Atorvastatin - 10mg od with high cholesterol myopathy, rhabdomyo- | alcoholics, pregnancy Simvastatin - 20-40mg od iysssimpotence Anti-platelets 4 vascular events Recent/active ASA ~ 80-162mg od bleeding Clopidogrel ~ 75mg od Glintolerance or ASA Warfarin ~ 1-10mg od allergy 8 NAC OSCE | A Comprehensive Review Atrial Fibrillation 1. To control rate: + Inj Metoprolol 5 mg bolus 1V, followed by infusion at 0.05 mg/kg/min, increasing as needed 10.0.2 mg/kg/min. + Inj Diltiazem 20 mg bolus. Maintenance infusion of 5-15 mg/h. + Inj Verapamil 5-10 mg IV over 2-3 min, repeated once after 30 mins. + Tab Amiodarone (in case of heart failuze + Loading dose: 800 ~ 1600 mg P mg/day divided bid-tid. + Maintenance: 200 mg PO od. 2. To prevent thromboembolism: Assess with CHADS 2 score + No risk: Tab Aspirin 81-325 mg PO od. +L moderate risk: Tab Aspirin 81-325 mg PO od or Tab Warfarin 2-15 mg PO od to maintain INR23. + > I moderate risk or very high risk: Tab Warfarin 2-15 mg PO od to maintain INR 2-3. 3. To control chythm: + Tab Flecsinide 300-400 mg PO bolus dose, maintenance: 50-150 mg PO bid. (First choice) + Tab Sotalol 80-160 mg PO bid. ( Second choice) + Tab Amiodarone (in case of heart failuse + Loading dose: 800 ~ 1600 mg PC mg/day divided bid-tid. + Maintenance: 200 mg PO od. + Electrical Cardioversion: 100-360 joules. divided doses until response; till max 1000 divided doves until response; till max 1000 Congestive Cardiac Failure : Immediate management in the ER CAR-TREATMENT MNEMONIC Litasix + Oxygen by nasal cannula at 4 Titers per minute. M:: Morphine Sulphate + Inj Furosemide (Lasix) 10 mg IV stat. Me eacenn + Inj Moxphine sulfate IV 2-5 mg every 5-30 min pn. Bae rieageee + Sublingual Nitroglyccrin 0.3-0.6 mg qSmin up to 3 ti Position > 45 degrees + Position of patient > 45 degrees. D: Dopamine (indicated in cardiogenic shock and hypotension) Non pharmacological management of Heart Failure + Brercise : Regular physical activity + Salt restriction : symptomatic HF ~ 2-3g salt/day (14 tsp/day) no added salt in diet. HE with fluid retention : 1-2g salt/day (% tsp/day) + Fluid intake : 1.5/2L per day in patients with fluid retention or HF not controlled by diuretics. + Daily weight measurement, + Education, + Aggressive risk reduetion (BP, glucose, lipids) + Lifestyle modifications, influenza vaccination. Therapeutic Guidelines | Medicine Dyslipidemia 1. HMG CoA Inhibitors: + Atorvastatin : Tab Lipitor 10-80 mg qhs + Rosuvastatin :Tab Crestor 10-40mg ghs + S/E: GI symptoms, rash, pruritus, inereased liver enzymes, myositis. + CALs active liver disease, muscle disease, pregnancy. ibrates: increased TG (triglycerides) + Fenofibrate : Tab Lipidil 67-200 mg/d 3. Bile acid sequestrants : increased LDL. + Tab Colestipol 5-30g/day 4, Cholesterol absorption inhibitors: + Tab Ezetimibe 10mg /day. Lo af Total cholesterol/HDL_ HIGH (10yr CAD > 20%) Target LDL - <2.0 Target <4 MODERATE (0y¢CAD > 10-19%) _| Treat fLDL- >3.5 Treat it>5 LOW (1oye CAD <10%) ‘Treat if LDL- =5 Treat re ~~ High Risk : All with CAD, CVD, most diabetes cases & chronic renal disease, Hypertension Non pharmacological treatment : + Smoking cessation: smoking aggravates hypertension and remains the major contributor to cardiovascular disease in people under 65 years. + Weight reduction : Maintain BM1<27, particularly in patients with glucose intolerance + Alcohol restriction, + Sodium restziction <150mmol/day: Blood pressure risk factors Consider treatment if BP BP target Noviskfactors > 160/100 «140/90 Isolated systolichypertension _SBP>160 seP<140 Moderate-High riskpatient — >140/90 oo Diabetes or Renal disease = 130/80 <130/80 10 NAC OSCE | AComprehensive Review Commonly used anti-hypertensive drugs : a - a . - Drug c ‘indication Side Effect Diuretics Uncomplicated HTN, Diabetes | Rash, allergic rxn, pancreatitis, sexual Hydrochlorothiazide HCT ~ 12.5-25mg od with normal albuminuria, LH | dysfunction. HCT contraindicated in Spironolactone ~ 25-50mg od and isolated systolic HTN gout, Beta Blockers Stable angina, Ml, LVH, Fatigue, insomnia, 4 HR, impotence, Metoprolol ~ 50mg bid or 100mg SR od uncomplicated HTN <60 years, | dizziness. C/ ~ asthma/COPD, 2/3" Propranolol ~ 80mg bid degree heart block, uncompensated HF Atenolol ~ 50-100mg od severe PAD ACE Inhibitors Heart failure, diabetes, post Ml, | Cough, loss of taste, rash, angioedema, Ramiprit ~ 10mg hs uncomplicated HTN, LVH, prior | renal failure, | BP Te a 7 ree CVAITIA, renal disease, all C/A ~ b/lrenal artery stenosis, Hx of ‘peed ona regnanc) Captopril- 25-50™mg bid coronary artery disease pts angioedema, pregnancy ‘Angiotensin Il Receptor Blockers Diabetes, uncomplicated HTN, Fatigue, headache, rash, angioedema, Losartan- 25-50mg od ‘solated systolic HTN, LVH, 4 BP, K+, pancreatitis. Valsartan ~ 80-160mg od patients unable to tolerate ACEI. C/I ~ b/l renal artery stenosis, Hx of Candesartan - 8-16mg od angioedema, pregnancy Calcium Channel Blockers Uncomplicated HTN, LVH, Angina, | Dizziness, headache, rash, edema, ‘Amlodipine ~ 2.5-10mg od Isolated systolic HTN, diabetes | gingival hypertrophy, worsen HF Nefidipine- 10mg tid without nephropathy CA ~ hypotension, recent Mi with Verapamil- 40-80mg tid pulmonary edema, sick sinus, Dittiazem ~ 30-60mg tid syndrome, 2"/3rd AV block Methyldopa ~ 125mg bid to 500mg qid First-line for hypertension in _| Sedation, dry mouth, hepatotoxic, pregnancy lupus like symptoms. Infective Endocarditis Prophylaxis + Inj Ampicillin 2g 1V q4h x dweeks. + Inj Gentamicin Img/kg LV q8h x4 weeks + Prophylaxis: Dental/respiratory/esophageal procedure: Tab Amoxicillin 2g PO. 30-60 min prior; ‘Tab Clindamycin 600mg PO, if allergic to penicillin, Rheumatic Heart Disease (RHD) + Tab Erythromycin 500 mg tid PO x 10days, * Tab Penicillin VK 500 mg PO bid x 10 days. Therapeutic Guidelines | Medicine u 2. Dermatology Acne Mild : <20 comedones (whiteheads/blackhcads) or <15 inflammatory papules, or a lesion count <30 Moderate : 15-50 papules and pustules with comedones, cysts are sare, lesion count ranges from 30-125 Severe : Primarily nodules and cysts,also present are comedones, papules and pustules, scarring is present, lesion count >125 1 Benzoyl Peroxide (Antibacterial/Keratolytic) Indication: 1" tine _S/E scontact dermatitis, 2 | Dose: apply to entire afected area ghs or bid medication for mild- dryness, erythema, burning He moderate acne. &pruritis {Tretinoin (Retinoid) Tline treatment for mild- | S/E erythema, dryness, L | Dose: qhs, apply 30-45 minutes after wash moderate comedones acne. _ burning, photosensitivity | Oral antibiotics Indicated for moderate-_S/E: Gl upset, nausea, Tetracycline - initial 500mg bid then 250-500mg od | severe acne. vomiting, candidiasis. Doxycycline - 100mg od ‘Acne on chest, back & (CAsliver disease Erythromycin - initial 500mg bid then 250-500mg od _| shoulders $ Combined Oral Contraceptive Females with moderate- C/I: Smoking, migraine with J Diane 35/Yasmin/Alesse: od x21 days, 7 days severe acne + seborrhea + aura, seizures 1 offfeycle hirsutism, late onset acne tsotretinoin Severe nodulocysticacne, | Teratogenicity : ocular 1 Accutane :0.5-1mg/ka/day x 16-20 weeks acne with scarting, failure to effects ~ conjunctivitis, © | important : Tests for pregnancy 30 days prior to respond to other treatments | night vision, premature starting Accutane, before each refill. Patient has to epiphyseal closure, 1 LFTs, sign an informed consent. pseudomotor cerebri, ‘mucocutaneous effects, myalgias. Photosensitivty. 1 of ABCs , consider the need for early intubation if airway is compromised. + Humidified 02 if any suspicion for inhalational injury. + Oxygen 100% if known carbon monoxide exposure of fire in an enclosed space. (Half life of hemoglobin will drop from 330 to 90 mins). + Establish IV access. + Fluid resuscitation : Parkland formula 4ml g/l + Nasogustrie tube drainage for ileus, + Bladder catheterization to monitor urinary output, minimum ImL/kg/he. axis : 0.5 ml. tetanus toxoid IM in previously immunized and 250 units‘T1G IM if burn, % over 8 hours and rest over 16 hours + Tetanus prophy d, unimmuniz 12 NAC OSCE | A Comprehensive Review Psoriasis ‘Topical Preparations : 1. Topical Corticosteroids + High Potency’Topical Steroids (Usually indicated) + Very high potency: e.g, Clobetasol (Temovate) + High potency: e.g, Fluocinonide (Lidex) + Low Potency Topical Steroids (Alclometasone dipropionate) usually indicated in + Genitals + Maintenance’Therapy 2, Vitamin D based topicals + Caleipotriene (Dovonex) + Used in combination with Topical Corticosteroids 3. Retinoid based topicals = + Tazarotene (Tazorac) ‘More irritating than Caleipotriene 4, Immunosuppressant based topicals : * Tacrolimus 0.1% or Pimecrolimus 0.1% ercams Effective in facial and intertriginous Psoriasis 5. Adjunctive agents in combination with above : + Topical Salicylic Acid (Keratolytic Agent) 6. Poorly tolerated topicals (use Calcipotriene instead) . + Historically used with UVB light exposure + Anthralin (Anthra-Derm) + Cou Tar (eg. Zetar) Ultraviolet light + Risk of non-Melanoma skin cancer + Protocols + Ultraviolet B exposure alone + Ultraviolet A exposure with psoralen (PUVA) Increased risk of non-Melanoma skin cancer Systemic agents (most are for higher risk) + Immunosuppressants + Etretinate + Cyclosporine + Methotrexate (unclear efficacy) ‘Therapeutic Guidelines | Medicine B + Biological agents + Tumor necrosis factor (TNF) receptor blockers Etanercept (Enbrel) Infliximab (Remicade) + Other mechanisms Alefacept (Amevive) izumab (Raptiva) + Thiazolidinedione (Avandia, Actos) ~ experimental + Appears effective in Psoriasis even in non-diabeties + Only small trials support to date Cellulitis + Cause: B Hemolytic Streptococeus , Staphylococcus + ‘Treatment: ‘Tab Cloxacillin 500mg PO qid x 10-14 days If patient is allergic to penicillin : Tab Cephalexin 500mg PO qid 10-14 days OR “Tab Clindamycin 300mg PO gid x 10-14 days Pediculosis + Permethrin 11% - wash hair with regular shampoo, then apply permethrin and leave for 10 mins then rinse + Pyrethrins with piperonyl butoxide + Lindane 1% C/L in neonates, young children and pregnant women, causes neurotoxicity + Wash all clothes and linen in hot water, then machine dry. + Permethrin 5% ~ massage into all skin areas, from the top of the head to the soles of the feet, leave for ‘8-14 hours then wash off. * — Crotamiton 10% + Scabene (aerosol spray) + Lindane : used only if allergic to permethrin, + Treat family and contacts. + Wash all clothes and linen in hot water, then machine dry. Tinea Cruris/Pedis (Jock itch/Athlete’s foot) + Clotrimazole 1% cream apply bid + Ketoconazole 2% cream apply bid 14 NAC OSCE | A Comprehensive Review 3. Endocrinology ‘Diabetes Mellitus Blood glucose target |AIC q3-6 months Normal range<6 Target 4-7mmol/t Normal range 4-6mmol/L Fasting plasma glucose Post prandial blood glucose 2h Target 5-10mmoV/L Normal range 5-8mmoV/L Approach to management of diabetes mellitus 1. Lifestyle modification & patient education 2. Oral hypoglycemic monotherapy + Biguanides (Metformin) ~ 250-500mg PO bid-tid (if obese or overweight) + Sulfonylureas (Glyburide) ~ 80mg PO bid + Thiazolidinecione (Pioglitazone) ~ 15img PO od; Rosi + Alpha glucosidase inhibitors (Acarbose) - 50mg PO tid 3. Oral combination therapy (2 agents often needed; after 3 years 50%, after 9 years 75%) 4, Tasulin therapy +/- oral hypoglycemics rvone— 4mg PO od + Fluid replacement + Initial : Give 1 liter NS bolus over fist 45 minutes, repeat fluid bolus until shock corrected. + Next : Replace first 50% volume deficit in first 8 hours, use Normal Saline or Lactated Ringers. Replace remaining 50% deficit over next 16 hours, use DS 1/2 NS at 150-250 ml per hour. + Insulin (Hypokalemia must be corrected prior to Insulin) » Initial i. Give IV bolus of 0.15 units/kg ii, Start 0.1 units/kg/hour Insulin Drip + Maintenance i. Anticipate Serum Glucose drop of 50-70 mg/dV/hour + Tfinadequate drop, then increase drip @) Tnerease Insulin Infusion rate by 50-10% b) Continue at increased rate until adequate ii, When Serum Glucose <200-250 mg/dl a) Keep Serum Glucose at 150 t0 200 mg/dl b) Decrease rate by 30% (t0 0.05 units/kg) or 9 ontinue Insulin Drip and start SC dosing Therapeutic Guidelines | Medicine 15 + Potassium Do not administer Insulin until potassium >3.3, + Give KC1 40 mE/hour IV un + Serum Potassium 3.3 to 5.0 mEq/L. i. Standard replacement: 20-30 mEq per liter + Serum Potassium >5.0 mEq/L. i. Do not administer any potassium i, Monitor every 2 hours until <5.0 corrects ications ABG pl < 6.9 t0 7.0 after initial hour of hydration ii, Other contributing factors + Shock or Coma : + Severe Hyperkalemia | Bipperthyroidiom + Tab Propylthiouracil(PTU) 100 mg PO tid, to max 150 mg 6-8 hours, + Tab Methimazole 10-30 mg PO od. + Medications associated with Hyperthyroidism: intake cess Thyroid horm Dietary lodine Amiodarone Hypothysoidiem + ‘Tab L-Thyroxine 0.05-0.2 mg/day + Medications associated with Hypothyroidism: i. Inorganic iodine ii, Iodide iii, Amiodarone iv. Lithium Hyperprolactinemia + Tab Bromocriptine 1,25-2.5 mg PO od, increase by 2.5 mg/day q3-7days to max 15 mg/day. + Tab Cabergoline 0.25 mg PO twice weekly, may increase by 0.25 mg q4weeks up to max Img twice weekly. 16 NAC OSCE | A Comprehensive Review Medications causing hyperprolactinemia ) ») 9 @ 2 f) 2 n) i ? » » m) n) °) P) Benzodiazepines Buspirone MAOT SSRI TCA Valproie acid Methyldopa Verapamil Atenolol Danazol Estrogen Depo-Provera OCPs Metoclopromide Amphetamines Cannabis Impotence "Tab Sildenafil 25-100mg per dose, to take half an hour to 4 hours prior to intercourse. S/E: flushing, headache, indigestion C/T: don't take with Nitrates: 4. Gastroenterology Acute Gastroenteritis Perioperative for 24hrs Inj Ampicillin 1-2¢ IV q4-6h. Inj Flagyl 500mg IV bid. Inj Gentamicin 3-Smy/kg/day q8h (monitor creatinine levels). NPO ‘Tab Flagyl 500 mg PO bid 5 days. E.Coli (Traveler's diarrhea) ‘Tab Ciprofloxacin 500 mg PO bid x 3 days. an “Tab Norfloxacin 400 mg PO bid x3 days. Cayptespaidium Oral rehydration solution | Giardia lamblia Therapeutic Guidelines | Medicine v7 + NPO + Inj Flagyl 400 mg TV gh + Inj Meperidine 75-100mg IV q2-3h + IVE + NG tube + Replace calcium ‘Crohn's s Discase 1, Mild to moderate + Tab Mesalamine 800 mg PO tid. Maintenance dose 3.2 ~4g per day: + Tab Sulfasalazine 250 mg per day and increase up to 2 g per day, Maintenance dose is 500- 1000 mg PO gid with food. 2. Moderate to severe: + Tab Prednisone 40 mg PO gid x 8-12 weeks and taper gradually. + Tab Azathioprine 2-2.5 mg/kg/day. Used for maintenance while tapering corticosteroids, Diverticulitis, + Inj Flagyl 500mg IV bid + Inj Ciprofloxacin SOOmg IV bid. Helicobacter Pylori 1. HP-PAC (7 blister pack) 7-14 days + Tab Lansoprazole 30mg PO bid + + Tab Clarithromycin 500mg PO bid + + Tab Amoxicillin 1g bid LINE Quadruple : 14 days + Tab Lansoprazole 30mg PO bid + Tab Flagyl 500mg PO bid + Tab Tetracycline 500mg bid * Tab Bismuth 525mg PO qid 18 NAC OSCE | A Comprehensive Review HepB ps xpe oe 1. Known HBsAg Positive Source: i. Unvaccinated exposed patient: + Hepatitis B Immunoglobulin (HBIG) 0.06 ml/kg and + Hepatitis B Vaccine 0,1and 6 months. ii, Exposed patient with known response to vacel + No teatment. iii, Exposed patient with known failed response to vaccine: + Patient has not yet completed second 3-dose series: + Hepatitis B Immunoglobulin (HBIG) 0.06 ml/kg and + Hepatitis B Vaccine (complete second 3-dose series) + Patient has completed two prior 3-dose series: + Hepatitis B Immunoglobulin (HBIG) 0.06 mi/kg + Second Hepatitis B Immunoglobulin dose. iv. Exposed patient with unknown response to vaccine: + Test for Antibody to HBsAg + Adequate Antibody (HBsAg Positive): No treatment + Inadequate Antibody (HBsAg Negative) + Hepatitis B Immunoglobulin (HBIG) 0.06 mV/kg and + Hepatitis B Vaccine booster dose: 2. Known HBsAg Negative Source: i, Administer Hepatitis B Va ii, No treatment otherwise needed. 3. Unknown HBsAg Source Status: i. Unvaecinated exposed patient + Hepatitis B Vaccine Series ji, Exposed patient with known response to vaccine + No treatment iil, Exposed patient with known failed response to vaceine + Treat source as HBsAg positive if high risk iv. Exposed patient with unknown response to vaccine + Test for Antibody to HBsAg + Adequate Antibody (HBsAg Positive): No treatment + Inadequate Antibody (HBsAg Negative) ‘+ Hepatitis B Vaccine initial and booster dose + Recheck titer in 1 to 2 months 4. Infant with ITBsAg Positive Mother: j i. Hepatitis B Immunoglobulin (HBIG) 0.5 ml within 12 hours of birth. i ii, Hepatitis B vaccine: Dose 1 within 12 hours of birth, Dose 2 at age 1 months, Dose 3 at age Series if unvaccinated 6 months, iii, Repeat HBsAg and HbsAb at 9 months & 15 months, Therapeutic Guidelines | Medicine 19 Reptic ulcer disease + Tah Omeprazole 20mg PO od. + Tab Ranitidine 150 mg PO bid. ‘Dleerative Colitis + ‘Tab Sulfasalazine 250 mg per day and increase up to 2 g per day. Maintenance dose is 500-1000 mg PO gid with food. + Tab Mesalamine 800 mg PO tid, Maintenance dose 3.2 ~ 4g per day. + Rectal suppositories preferred for proctitis. ‘Acute Cholecystitis (Perioperative) + Inj Cefaolin 0.5-1.5mg IV q6h + NPO + IVE + NG Tube 5. Hematology Anemia + Toon Deficiency Anemia : Tab Ferrous fumarate(Palafer) 300 mg PO gd OR ‘Tab Ferrous Sulfate 325 mg PO qd “Tab Ferrous Fumarate 300mg PO gd + Tab F 6. Infectious Diseases Prophylaxis for opportunistic infections in HIV patients + Pneumocystis cariniis CD4 counte 200 cells/mm” or oral candidiasis. ‘Tab TMP/SMZ DS PO OD till CD4 counts + Toxoplasma gondii: IgG antibody positive and CD4 count < 100 cells/mm? th, TMP/SMZ DS PO OD till CD4 counts rises. + Mycobacterium tuberculosis: Mantoux > 5 mm in immunocompromised or contact with active TB. ~ Tab Isoniazid 300 mg PO OD x 9 months along with ~ Tab Pyridoxine 50 mg PO OD. + Mycobacterium avium complex: CD4 counts < 50 cells/mm’. - Tab Azithromycin 1200 mg PO once a week. ~ Tab Clarithromycin 500 mg PO once a week. + Varicella zoster virus: Recent exposure to chicken pax or shingles ~ Varicella zoster immune globulin within < 96 hours of exposure. 20 NAC OSCE | A Comprehensive Review ‘HIV post exposure prophylaxis + Start within hours of exposure (under 24 to 48 hours). + Triple Therapy for 4 weeks: 1, Fisst two medications: AZT and 3TC (or Combivir) i. Tab Zidovudine (AZT) 300 mg PO bid and ii. Tab Lamivudine (3TC) 150 mg PO bid. 2. Third medication (choose one): i. Tab Indinavir (IDV) 800 mg PO tid or ii, Tab Nelfinavir (Viracept) 750 mg PO tid or iii, Tab Efavirenz 600 mg PO qhs. + Obtain baseline labs to monitor for adverse reaction: 1. Pregnancy Test Complete Blood Count with differential and platelets Urinalysis Renal Function ‘Tests Liver Function Tests weer Malaria 1, Treatment for active infection: i, Tab Chloroquine 1 g PO stat, then 500 mg PO 6-8 hours later, then 500 mg PO at 24 hours (& 48 hours after initial dose. ji, Tab Mefloguine 1250 mg stat dose. iii, ‘Tab Primaquine 15 mg base PO od x 14 days. 2. Chemoprophylaxis: i. Tab Chloroquine 500 mg PO once a week ii, ‘Tab Mefloquine 250 mg PO once a week. Pulmonary tuberculosis 1, Initiation Phase: Tab Rifampin 600 mg +’Tab Isoniazid 300 mg + Tab Py 2 months. . Continuation Phase: Tab Isoniazid 300 mg + Tab Rifampin 600 mg for 4 months. 3. Add Tab Pyridoxine (Vit B,) 50 mg PO OD. inamide 2 g for Rabies a er Wash wound with soap and water. + Human Rabies Immunoglobulin 20 TU/kg IM stat and half dose into the wound. + Rabies vaccine 1 ml IM on days 0, 3,7, 14,28. + Inform Public Health. + Capture animal & observe x 10 days, then examine brain for negri bodies. ‘Therapeutic Guidelines | Medicine 21 ‘Tetanus Prophylaxis: Based upon Tetanus immunization stacas ~ ae Clean, minorwounds | _Allother History of tetanus immunization oe SSeS Td orTdap* 0.5mi | Tig** 250U | Td or Tdap* | Tig ‘@ecerain or <3 doses ofan immunization Yes Yes | > 3dosesreceived in animmunization Not No ™ adult-type combined tetanus and diphtheria toxoids or a combined preparation of diphtheria, tetanus and acellular pertussis. Ifthe patient is <7 years old, a tetanus toxoid-containing vaccine Is given as part ofthe routine childhood emunization. * Tetanus immune globulin, given at a separate site from Td (ot Tap) ¥es,if> 10 years since last booster. 5 Yes if > 5 years since last booster, More frequent boosters not required and can be associated with increased adverse cerents. The bivalent toxoid, Td, snot considered to be significantly more reactogenic than Talone and Is recommended ‘© Use in this circumstance. The patient should be infornied that Td (or Tdap) has been given, 7. Neurology ‘Seizures 1. Acute Management: © Inj Diazepam 5-10mg IV q2-3mins till seizure stops. + Inj Phenytoin 20mg/kg IV at 50mg per min, * Inj Phenobarbital 20mg/kg IV at 50-7Smg/min + Ifall fails then rapid sequence intubation. ieee 2. Primary Generalized & Partial seizures: ee + Tab Phenytoin: Loading 300mg PO q4h x 3 doses, | 0:Osteomalacia then 300mg PO qhs. ' peer ci folic =a + Tab Valproate: Loading 15mg/ky/day, increments by NiNewopatues vertgo, 1Omg/kg/day qweeldy, tll seizures are controlled. ataxia, headache * Tab Carbamazepine: Start 100-200mg PO od-bid, ‘ increments by 200mg/per q2d, if needed till max : = 800mg-1200mg per day. 3, Absence Seizures: + Tab Ethosuximide 500mg PO daily in divided doses, increments by 250mg/day q4-7d prn till max 1500mg per day. Meningitis * — Investigations : CT then L + Empirical adule antibiot Inj Ceftriaxone 2g 1V q12h Inj Dexamethasone 10mg qh IV x 4 days for pneumococcal meningitis Meningococcal: give contacts Tab Rifampin 600mg PO ql2h x 4 doses CSF analysis, blood C&S, neurology consult 3" generation cephalosporins + vancomycin + ampicillin 22 NAC OSCE | A Comprehensive Review CSF Findings : Normal or mildly increased Normal Turbid Gear 018-045 | 1 <1 2535 <22 ___ Normal Normal 60-50% Postive Normal 06 oa 206 <3 -_>500 <1000 90% PMIN Monocytes 10% have >90% PMN, 30% have >50% PMN Fibrin web 0-05 1625 a <04 100-500 Monocytes + ‘Tab'Tripran and ‘Tab Prednisone at the beginning of the cycle and prophylactic treatment with Tab Lithium(300-600mg daily initially chen monitor serum levels) + Dihydroergotamine nasal spray 4mg per 1 ml. One spray each nostril and repeat q15mins. Migraine 1. Mild ~ Moderate + NSAIDS + Tab Ibuprofen 200mg tid + Tab Aspirin 600mg PO 4h 2. Moderate ~ Severe + TRIPTANS + Tab Sumatriptan 25mg PO & repeat q 2hrs prn + Tab Mecoclopramide 10mg PO stat 3. Prophylaxis: + Tab Propranotol 60mg PO daily + Tab Amitriptyline 10-25mg PO qhs. “Tecate Therapeutic Guidelines | Medicine 23 Myasthenia Gravis 1. Anticholinesterase (Cholinergic) + Tab Mestinon (Neostigmine and Pyridostign 2. Immunosuppressive therapy + Tab Prednisone: Start at 20 mg qd, increase gradually by 5 mg every 3 days to 60mg, Continue for 3 months or until clinical improvement stops or declines. Taper gradually to every other day . + Tab Azathioprine (Imuran) 2 mg/kg/clay. Effective when given with Prednisone. Effect not seen for 6 months or more, Monitor CBC and LF'TS 3. Plasmapheresis (Plasma Exchange) and IV Ig: Indicated for emergent worscning/crisis. Response rate: 70%. + Tab Carbidopa/L evodopa 25/100 mg PO bi + Tab Bromocriptine 1.25 mg PO bid. + ‘Tab Pergolide 0.05 mg PO od, titrate q2-3 days to the desired effect. Maintenance dose is 3-6 mg/day in divided doses. ‘Tab Premipexole 0.125 mg PO tid, increase to 1.5 - 4.5 mg/day in divided doses. + Tab Ropinirole 0.25 mg PO tid, increase weekly to max dose 24 g/day. Amantadine 100 mg PO od to max 100 mg PO gid. Tab Selegiline 5 my PO bid. fab Benztropine 0.5-6 mg/day PO in divided doses. ‘Tab Entacapone 200 mg given concurrently with Carbidopa/Levodopa. : 60-120 mg q3-4h. «id, inerease as needed to max 200/2000me/day. 8. Otolaryngology Acute Sinusitis + Tab Amoxicillin 500mg tid PO x 10 days, + Decongestant: Tab Sudafed 60mg PO qoh + Nasal saline. Acute Pharyngitis + Group A Hemolytic Strep: Tab Penicillin V 300mg PO tid x 10days + Penicillin allergie:'Tab ythromyein 500mg tid x 10 days 24 NAC OSCE | A Comprehensive Review 9. Pulmonology Asthma 1. Intermittent Asthma: Short acting beta-agonist ~ Salbutamol (Ventolin) Inhaler 1-2 puffs q4-6h pn. 2. Mild Intermittent Asthma: + Long acting beta agonist ~ Salmeterol Inhaler 1-2 puffs bid. + Inhaled steroids: i, Fluticasone (Flovent) 2-4 puffs bid. ii, Budesonide (Pulmicort) 2 putts bid. iii, Beclomethasone (Vanceril) 1-4 pufis (40ug) bid or 1-2 puffs (80pg) bid. 3. Moderate Persistent Asthma: + Inhaled steroids: Fluticasone (Flovent) 2-4 puts bid. Budesonide (Pulmicort) 2 puis bid. iii, Beclomethasone (Vanceril) 1-4 pulls (40pg) bid or 1-2 pufls (80yx) bid. + Long acting beta agonist ~ Salmeterol Inhaler 1-2 puffs bid. + Leukotriene Receptor Antagonist: + ‘Tab Montelukast 10 mg PO qhs. + Tab Zileuton 600 mg PO gid. 4. Severe Persistent Asthma: + High dose Inhaled steroids. + Long acting beta agonist. + Leukotriene Receptor Antagonist. + Systemic Steroids: i, Tab Prednisone 2 mg/kg/day PO (max 60 mg/day). ii, Inj Methylprednisolone (Depo-medrol) 2mg/kg IV, then 0.5 mg/kg qéh x Sdays. Acui ition of COPD + Admit with nasal O,.Keep saturation between 88-92% . If silent chest/GCS < 8 or decreased LOC then intubate. + Elevated bed > 43 degrees. + WE + MDI: 8 puffs of Ventolin (Salbutamol) alternate with 8 putts of Atrovent (Ipratropium) back to back every 20 mins 3 times. + Nebulizer : 2ce Ventolin + Ice Atrovent in 3ce NS q20 inins x 3 mes. + Inj Hydrocortisone 125mg, IV stat, if severe. “eftriaxone 1-2 g IV q24h along with ‘wobactam 3.375 g IV q6h. + Inj Methylprednisolone 2mg/kg IV, then 0.5 mg/kg. qoh x 5 days. Therapeutic Guidelines | Medicine 25 Community Acquired Pneumonia 1. Outpatient management: + Tab Doxyeycline 100 mg PO bid x 7-10 days. + Tab Erythromycin 250 ~ $00 mg bid x 7-10 days, + Tab Azithromycin 500 mg PO od x 5 days. + Tab Levofloxacin $00 mg PO od x 7-10 days, ‘Tab Augmentin 500 mg/ 125 mg PO gh x Sdays. 2. Inpatient management: + Inj Ceftriaxone 1-2 IV bid along with + Inj Levofloxacin 500 mg IV od x 7-10 days. + Inj Azithromycin 50 mg IV over 1 hour od x 1-2 days. Pulmonary Embolism 1. Investigations + V/Qscan, spiral CT or D-dimer (if unlikely Wells’ seore < 4) + CBC, INR, PTT, BUN, creatinine, ALT, AST. 2. Management: Initiation + Start Warfarin (Coumadin) concurrent with Heparin. + Contraindicated in pregnancy. (If contraindicated may put IVC filter) + Start Tab Warfarin at 5 mg PO daily on Day 1-2 and Heparin 5000 U TV bolus followed by continuous infusion 20 U/kg/hour, titrate to INR 2-3 then stop heparin within 24 hours, + Check INR in 3-5 days, + ‘Therapeutic INR: 2.0 10 3.0 1U, + Osygen, and if pain give morphine or NSAID. 3. Management: Duration of Anticoagulation © Very low risk: 6-12 weeks + Symptomatic isolated calf vein thrombosis. + Low risk patient: 3-6 months Reversible thromboembolism risk (transie + Upper extremity Deep Vein Thrombosis. + Moderate risk patient: 6-12 months + First idiopathic DVT or PE. + High risk patient: 12 months or lifetime Anticoagulation + Recurrent DVT or PE or'Thrombophila risk such as post-operative PE). 26 NAC OSCE | A Comprehensive Review 10. Rheumatology Osteoporosis Tab Calcium (1500mg/day) and Tab Vitamin D (800 TU/day) intake in diet or as supplements. + Bisphosphonates: Alendronate, Risedronate or Raloxifene. + Hormone Replacement Therapy + Calcitonin + Recombinant Parathyroid Hormone + Lifestyle modifications: Weight bearing exercises, smoking and alcohol cessation. Osteoarthritis + Tab'Tylenol 500 mg PO tid + Tab Ibuprofen 200-600 mg PO tid. + Tab Naproxen 125-500 mg PO bid. + Tab Celecoxib 200 mg PO od. + Other treatment: + Tab Acetaminophen +"Tab Codeine, + Tntra-articular corticosteroid injection. + Ingr-articular hyaluron injection. + Topical NSAIDs. + Capsaicin cream, + Glucosamine sulfate, Rheumatoid Arthritis 1. First Choice: + Tab Naproxen 500 mg PO bid. + Tab Ibuprofen 300-800 mg PO qid. + Tab Indomethacin 25-50 mg PO bid or id, 2. Analgesies: Tib Acetaminophen 500 mg PO tid pen, 3. Corticosteroids: given intra-articular i, Small Joints: + Inj Hydrocortisone 8-20 mg + Inj Methylprednisolone 2-5 mg, + Inj Betamethasone 0.8 - 1.0 mg, ii, Large Joints: + Inj Hydrocortisone 40, 100 mg. + Inj Methylprednisolone 10 - 25 mg. + Inj Betamethasone 2 - 4 mg. Therapeutic Guidelines | Medicine 27 4, Disease Modifying Anticheumatic Drugs (DMARDs): Start within 3 months of diagnosis to reduce discase progression. i, Mild disease: + Tab Hydroxychloroquine 200 mg PO bid. * Tab Sulfasalazine 500 mg PO bid to tid, ii, Moderate disease: + Tab Methotrexate 10-15 mg PO once weekly, then increase to 20 mg PO once weekly. + Combination therapy: + Methotrexate + Sulfasalazine + Hydroxychloroquine. + Methotrexate + Cyclosporine, + Methotrexate + Etanercept (biological DMARD), iii, Biological DMARDs: used in persistent disease: + Etanereept $C, + Infliximab IV. + Anakinra SC. + Adalimumab SC. + Abatacept IV, + Rituximab IV. + If Corticosteroids are used for> 3 months, do baseline DEXA and start bisphosphonate therapy. + SE of Corticosteroids: Osteoporosis, cataracts, glaucoma, peptic ulcer disease, avascular necrosis, hypertension, increased infection rate, hypokalemia, hyperglycemia, hyperlipidemia, * C/I to Corticosteroids: Active infection, hypertension, diabetes mellitus, gastric ulcer, osteoporosis. 1. Acute Gout: SAIDs:’Tab Indomethacin 25-50 mg PO tid x 10-14 days. ‘Tab Naproxen 500 mg PO bid x 4-10 days. ‘Tab Colchicine 0.6 mg PO q1h till pain relief (max 4-6 doses), then bid x 3-5 days, iv. Systemic Steroids: (rule out Septic Arthritis) + Inj Methylprednisolone 40 mg IV single dose + Inj Depo-Medrol 80-120 mg IM single dose. + Oral: Tab Prednisone 40 mg PO od x Sdays, then gradually taper the dose, v. _Intra-Articular Corticosteroid: used in lange single joints & refractory cases. + Inj Betamethasone 7 mg or Inj ACTH 40-80 IU. 2, Recurrent Gout: «ut for 3-6 months. i. Over producers: Tab Allopurinol 100-300 mg/day PO. ii, Under-excreters: Tab Probenecid 250 mg PO bid (max:1500 mg bid) or Tab Sulfapyrizine 50 mg PO bid (max: 1000 mg bid). iii, Concurrently start with Tab Colchicine 0.6 mg PO bid x 3-6 months. 28 NAC OSCE | A Comprehensive Review ‘Temporal arteritis + Start high dose Tab Prednisone 60 mg PO od until symptoms subside and ESR normal + Then 40 mg PO od for 4-6 weeks + Then taper to 5-10 mg PO od for 2 years (relapses occur in 50%) if treatment is terminated before 2 years). Treatment does not alter biopsy results if the sample is taken within 2 weeks. + “Monitor ESR regularly. + If visual symptoms are present, or develop during treatment, the patient is admitted and given Inj Prednisolone 1000 mg IV q12h for lays. Polymyalgia Rheumatica Management 1, General measures * Consider concurrent Temporal Arteritis (See above) + NSAIDs 2. Prednisone (key to management) + See Corticosteroid Associated Osteoporosis + Efficacy: 90% response Dramatic improvement in first 48 hours If no response to steroids ~ reconsider diagnosis Reconsider diagnosis Consider Methotrexate + Polymyalgia alone Dose: 15-20 mg PO qd + Polymyalgia with Temporal Arteritis Dose: 40-60 mg PO qd Symptoms and signs remit within 1 month Decrease dose by 10% each week after improvement + Course + Initial: Maintain starting dose for 1 month + First steroid taper (depends on clinical response) ‘Taper by 2.5 mg per month down to 10 mg/day then Taper 1 mg per 4-6 weeks down to 5 to 7.5 mg/day + Final steroid taper Indicated when symptom five for 6-12 months Do not taper until sedimentation rate normalizes ‘Taper by 1 mg every 6-8 weeks until done + Anticipate 2-6 year Relapse common in first 18 months of steroid use Patients off steroids at 2 years: 25% e of steroids Therapeutic Guidelines | Medicine 29 Fibromyalgia 1. ANTIDE! SAI * Assists with loe: + Does not affect ‘Tender Points 2. Tricyclic Antidepressants + Amitriptyline (Wlavil) i, First week: 10 mg PO qhs ii, Next three weeks: 25 mg PO ghs iii, Later: $0 mg PO ghs + Nortriptyline (Pamelor) 3. Novel Antidepressants + Venlafaxine (Effexor) + Duloxetine (Cymbalta) 4, Selective Serotonin Renptake Inhibitors (SSRI) + Combination: Fluoxetine and Amitriptyline Benefits pain, stiffness and sleep Scones + Gonococcal: Inj Ceftriaxone 1g TV q24h x 2-4 days, then switch to Tab Ciprofloxacin 500 mg PO bid x 7 days + Non-Gonococeal: Inj Naficillin 2g IV q4h x 2 weeks, then switch to Tab Ciprofloxacin 500 mg PO bid x 2-4weeks. 11. Urology/Nephrology Urinary tract infection (UTI) 1, Acute uncomplicated UTI; outpatient + Tab Bactrim DS PO bid x 3 days. * Tab Nitrofurantoin (Macrobid) 100 mg PO bid x S days. UTE: outpatient Ciprofloxacin 500 mg bid x 3 days. Norfloxacin 400 mg PO bid x 3 days. + Tab Ofloxacin 200 mg PO bid x 3 days. 3. Acute complicated UTL: inpatient * Inj Ampicillin 1-2 g IV q4-Gh and Inj Gentamicin 2mg/kg IV loading dose followed by 1.7 mg /kg q8h IV OD + Inj Ciprofloxacin 400 mg IV bid. * Switch to oral antibiotics upon improvement for a total course of 14-21 days. 2, Drug resi 30 NAC OSCE | A Comprehensive Review ‘Acute Pyelonephritis 1. Ourpatient management: For acute uncomplicated cases + Tab Ciprofloxacin 500 mg PO bid x 10 days. + Tab Gatifloxacin 400 mg PO daily x 10 days. + Tab Moxifloxacin 400 mg PO daily x 10 days. + Tab Levofloxacin 250 mg PO daily x 10 days. + Tab Augmentin bid x 14 days. + Tab Bactrim bid x 14 days. 2. Inpatient management: [V for 48-72 hours, then switch to oral agents. Total duration of treatment for 14 days. + Inj Ceftriaxone (Rocephin) 1-2 grams 1V 424 hours. + Taj Cofotaxime (Claforan) 1 gram LV q12 hours + Inj Ampicillin 2 x IV qoh with Inj Gentamicin 2mg/kg IV loading dose , then 1.7mg/kg sh. * Inj Piperacillin 3.375g LV q6h. 12. Emergency Medicine/Poisoning Acetaminophen Intoxication + Toxic level dose is more than 7.5 + Iavestigations : Monitor drug level stat and then q4h (Acetaminophen nomogram), LFT, INR, PTT, BUN, Creatinine, ABG, Glucose + Rx: Charcoal/Gastric lavage as per presentation ‘N-acetyl cysteine 140mg/kg PO, then 7Omg/kg q4h for 18 doses Alcohol withdrawal + Treatment: Inj Diazepam 10-20mg 1V Inj Thiamine 100mg IM then 50-100mg/day Fluid resuscitation with DSW 1-2mL/kg IV Allergic Revetion 1. Severe: Inj Epinephrine 0.3-0.5 mg SC/IM stat 2, Mild: Tab Benadryl 25-50mg PO q6hx 3d 3. Tab Prednisone 60mg PO od x 3d Abaphylnds + Epinephrine autoinjector (HpiPen) if available + Epinephrine 1V or ETT : Iml of 1:10,000 in adults + Inj Diphenhydrumine (Benadryl) 50mg IV or IM q4-6 bh + Inj Methylprednisone 50-100mg IV according to severity + Te wheezing or spasm present : Salbutamol via nebulizer. Therapeutic Guidelines | Medicine 31 Asrhythmias Arrhythmias duc to 2° degree and 3" degree heart block : Inj Atropine 0.5mg TV while waiting for transcutaneous pacing, ‘Transcutaneous pacing first (give Inj Midazolam 2mg for sedation) Admit for transvenous pacing Unstable patients (hypotensive systolic BP < 90, chest pain, SOB, altered mental status or “ARDIOVERT! unconscious) Stable patient Atrial fibrillation : either chemical cardioversion (Amiodarone) or electrical (Synchronized DC cardioversion) Ventricular tachycardia : DC cardioversion or Inj Lidocaine/Amiodarone 150mg IV over 10 mins, Ventricular fibrillation : Always defibrillate! Synchronized cardioversion not useful because there is no QRS complex to synchronize with PSVT : Valsalva or carotid massage (after checking for bruit), Inj Adenosine 6mg rapid IV push. Ifno response then Metaprolol, Dild ASA Intoxication Investigations : Drug levels, electrolytes, ABG, BUN, Creatinine Rx: Gastric lavage/Charcoal Alkalinize urine with DSW, KCl and NaHCO, ‘Aim : urine pH > 7.5 Diabetic ketoacidosis Estimated daily basal glucose requirement is 0.5U/kg, Investigations: Blood glucose, electrolytes, ABG, serum ketones, osmolar gap, anion gap, BUN, creatinine. Look of the cause : Urinalysis, blood C&S, chest x-ray, ECG. Monitor : Urine output, extra-cellular fluid volume, electrolytes, ABG, creatinine, capillary blood glucose and level of consciousness every 1-2 hours Management: Rehydration : NS 11/h in first 2 hours followed by 0.45% NS 500cc/h then switch to maintain blood glucose 13.9-16.6mmol/L to avoid rapid decrease of osmolality, K+ replacement : As acidosis is corrected, hypokalemia may develop. If K+ is 3.3-5.0 mmol/L, add KCI 20-30 mEq/L to keep it within this range. Correct acidosis : If pH < 7,0/hypotension/coma then give 3 amp NaHCO, (150mEq/L) Reduce blood glucose : Start Insulin therapy with 0,15U/kg bolus and maintain 0.1U/kg/h until acidosis and blood glucose resolve. “Treat underlying precipitant. 32 NAC OSCE | A Comprehensive Review Digoxin Intoxication + Investigations : Plasma digoxin/digitoxin levels, ECG, electrolytes, BUN, Cr ( levels > 2.6 indicate intoxication) + Rx: Treat arrhythmias (common with digoxin intoxication; vfib, vtach, conduction blocks) Gastric lavage / Charcoal (1g/kg) for ingestion NaHCO3 or glucose and insulin Ventricular tachycardia : Digibind 10-20 vials if dose unknown, Chronic toxicity : then Digibind 3-6 vials IV over 30 mins. Follow ECG, K” Mg’, Digoxin levels every 6 hours, + Systolic BP > 180mmflg and Diastolic BP » 120mmHg (with signs of acute organ damage) + Investigations : CBC, electrolytes, BUN, Creatinine, ABG, Urinalysis, CXR, ECG, BP in all four limbs, Fundoscopy, Cardiology consult. + 1st Line: Inj Sodium nitroprusside 0.3 meg/kg/min IV OR Inj Labetalol 20mg 1V bolus q 10 mins. + Aortic dissection : Sodium nitroprusside + Beta blocker (esmolol) + Catecholamine excess : Inj Phentolamine $-15mg TV q 5-15 mins + Mi/Pulmonary edema : Inj Nitroglycerin 5-20meg/min TV, increase by Smeg/min every 5 min dill symptoms improve. Hypoglycemia + Investigations: Baseline blood glucose, insulin and C-peptide, check glucose q15 mins until » Smmol/L, tient can eat/drink : give 15g carbohydrate if BG < 4mmol/L. (15g glucose tabs or % caps of juice or 3 spoons of sugar in water.) NPO : give 25g carbohydrate if BG < 4mmol/I. ( D50W 50cc IV push 1 amp OR DIOW 500ce TV OR glucagon 1-2mg IM/SC ) + Rx: If Methanol/Ethylene glycol intoxication + Investigations; CBC, electrolytes, glucose, methanol level. = Re: Ethanol 10mg/kg over 30 mins OR Inj Fomepizole 15mg/kg 1V over 30 mins. Therapeutic Guidelines | Medicine 33 Opioid Intoxication + Mental status effects includ sedation, decreased anxiety, a sense of tranquility and indifference to pain produced by mild-to-moderate intoxication. Severe intoxication ean lead to delirium and coma + Physiological effects include the following: Respiratory depression (may occur while the patient maintains consciousness) Alterations in temperature regulations Hypovolemia (truc as well as relative), leading to hypotension Miosis Soft tissue infection Increase sphincter tone (can lead to urinary retention) + Treatment IV glucose : 50% Dextrose 50ml Inj Nalaxone 0.4mg upto 2mg IV for reversal of opioid intoxication, Inj Thiamine 100mg IM stat & OD x 3days 02, intubation & mechanical ventillation Shock (Curdiogenie/Neurogenic) + Dopamine : 1-3mey/ky/smin is the renal dose; 4-10meg/ky/min is the inotropic dose + Dobucamine : 2.5-Smey/kg/min Sprain (Ankle) RICE + Rest + Ice: using bag of ice, apply during the day for $-20 mins every 2 hours. + Compression : Tensor bandage or special supports. + Elevation : Elevate the ankde as much as possible + Analgesics as needed. + Crutches if too painful to bear weight. + Investigations; CBC, clectrolytes, BUN, glucose, creatinine, INR/PTT if suspecting TIA, ABG, Non contrast urgent CT scan. + Treatments: NPO, Foley catheter, DVT prophylaxis, Neurology consult, Rule out contraindications for thrombolytic treatment. Urgent neurology consult. ‘Thrombolysis : TPA within 3 hours of symptoms Anti-coagulation : Low dose Heparin 5000 U bid, start Warfarin within 3 days, monitor INR/PTT Tfunable to thrombolyse or anti-coagulate then :'Tab ASA 50-325mg od or Tab Clopidogrel 75mg od BP control : decrease slowly, IV Labetalol (First line treatment) Bed rest, analgesics, mild sedation and laxatives, avoid hyperglycemia. , carotid doppler 34 NAC OSCE | A Comprehensive Review + Patients who present to the ED following psychotropic drug overdose with GCS <8 should undergo intubation at the earliest opportunity to prevent hypoventilation and aspiration pacumonia, tions : Drug levels, ECG, ABG, electrolytes, LFTs, RFTs. + Re: Activated charcoal 1gm/kg via NG Diazepam for seizures Wide ORS/Seizures : NaHCO, (1-2 mg/kg bolus dose and then 100-150 mEq in 11, 15/0.45% NaCl infused 100-200 mi/h IV) + Invest Upper GI Bleed ent with IVF, cross & type, 2 large bore IV cannulas. + Investigations : CBC, platelets, INR, BUN, creatinine, PT'T, electrolytes, LFTs * Management: NG tube, NPO, blood transfusion if needed, upper GI endoscopy Inj Octreotide SOmeg loading and 30meg per hour (for varices) SC/IV Inj Pantoprazole 50mg LV stat and SOmg q8h (gastric ulcer) Lower GI Bleed + Stabilize patient with IVE, eross & type, 2 large bore IV cannulas. + Investigations: CBC, platelets, INR/PTT, BUN, creatinine, electrolytes. + Management: NG tube, NPO, blood transfusion if needed, sigmoidoscopy, colonoscopy, angiogram (for angjodyspl: ‘Warfarin Intoxication + Treatment according to INR levels INR < 5: Stop warfarin, observation, sesial INR/PTT. INR 5-9: Ifno risk factors for bleeding, hold warfarin x 1-2 days & reduce maintenance dose. OR Vitamin K 1-2 mg PO, if patient at increased risk or FFP for active bleeding. top warfarin, Vitamin K 2-4 mg PO, serial INR/PTT then additional Vitamin K if needed or FFP for active bleeding. INR > 20: FFP 10-15ml/kg, Inj Vitamin K 10mg LV over 10 min, increase dose of Vitamin K (qh) if needed. INR 9-20: Therapeutic Guidelines | Medicine 35 13. Counselling Smoking cessation 1. Nicotine gums: 2mg if < 25 cig/day, dmg i> 25cig/day * 1 piece qt-2h for 1-3mths 2. Nicotine patch: + 21mg per day for 4 weeks + 14mg per day for 2 weeks + 7g per day for 2 weeks Nicotine inhaler: 6-16 cartridges per day upto 12 weeks 4. Bupropion(Zyban): + 150mg GAM x days, then 150mg bid for 7-12 weeks + Maintenance 150mg bid for upto 6 months. * General Stop smoking during second week of medication Stop Bupropion if unable to quit by 7 weeks Minimum of & hours between doses More is not better Swallow pills whole (not crushed, divided or chewed). Alcohol cessation Protocol: Aleohol Dependence CAGE Question ee + Lab markers C :Have you ever fet the need Serum Gamma glutamyl transferase or Je CUT downs your Carbohydrate deficient Transferrin es ae fe ANNOYED at criticism of Initial Management yourdrinking? * Tab Thiamine 100 mg PO qd G pha 2 oo tH nen «Tab Folate 1 my your drinking? re ae ie Relat E; Have you ever hada drink © Multivitamin gd first thing inthe morning + Treat Hypomagnesemia if present | (EYEOPENER)? + Seizure precautions | 2.Long-Term Abstinence Programs + Alcoholics Anonymous + Detoxification centers + Halfway House 36 NAC OSCE | A Comprehensive Review, 3. Adjunctive Medications for abstinence 1LFirst line (consider Naltrexone with Campral) + Tab Naltrexone Blocks Opioid receptors Decreases pleasure from Alcohol Dosing: 50 mg orally daily Eficetive in short-term, but not in long-term + Tab Campral (Acamprosate) Balances GABA and glutamate neurotransmitters Reduces ansiety from abstinence Dosing: 2 tabs PO tid 2.Second line agents to consider + Selective Serotonin Reuptake Inhibitors (SSRI) Consider especially if comorbid depression Provac often used, but other SSRIs effective © "Tbpitwisane Tepans) Decreases Alcohol use severity and binge drinking Improves well being, quality of life in Aleoholies 3.Agents to avoid + Antabuse “Taken 250 to 500 mg orally daily Not recommended due to risk and uncertain benefit + Delirium ’Tremens General Protocol (Requites ICU observation) + Tab Diazepam (Valium) Dose: 10-25 mg PO qh pm while awake Endpoint: until adequate sedation + Inj Lorazepam (Ativan) Dose: 1-2 my IV qh pr while awake for 3-5 days Endpoint: until adequate sedation + Librium (Chlordiazepoxide) Dose: 50 to 100 mg PO/IM/IY qth (max: 300 mg/day) Endpoint: until adequate sedation NOTES 38 NAC OSCE | AComprehensive Review Obstetrics & Gynecology 1, Semually Transmitted Infection a. Chlamydia Tab Azithromycin 1g PO stat or Tab Doxycycline 100mg PO bid x 7 days If pregnant: Tab Erythromycin 500mg PO tid x 7 days. Treat partner, Reportable disease. b. Gonorrhea: Inj Ceftriaxone 125mg IM stat + Tab Doxycycline 100mg bid x 7 days. If pregnant : Inj Spectinomycin 2y¢ IM stat ‘Treat partner, Reportable disease. . Syphilis: Primary, Secondary, Latent Syphilis (duration less 1 year ): Inj Benzathine Penicillin G 2.4 MU IM for 1 dose ‘Treat partnes, Reportable disease. If allergic to Penicillin: Tab Doxycycline 100 mg PO bid for 14 days. Late latent, Cardiovascular (duration over 1 year) Inj Benzathine Penicillin G 2.4 MU IM once a week for 3 weeks If Penicillin allergic : Tab Tetracycline 500 mg PO gid for 4 weeks or ‘Tab Doxycycline 100 mg PO bid for 4 weeks Neurosyphilis : Inj Aqueous Penicillin G 3-4 MU IM every 4 hours for 10-14 days. d. Genital herpes: First episode: ‘Tab Acyclovir 400mg PO tid x 10 days or ‘Tab Famciclovir 250 mg tid x 10 days or ‘Tab Valacyclovir 1 g bid x 10 days Recurrent: ‘Tab Acyclovir 400mg PO tid x 5 days or “Tab Famciclovie 120 mg bid x 5 days or “Tab Valacyclovir 500 mg bid x 5 days Suppression: if more than 6 episodes per year ib Acyclovir 400mg PO bid x 12 months Severe episode: Inj Acyclovir 5-10 mg/kg qBh x 5-7 days ¢. Genital warts (HPV): Local treatment with LIQUID NITROGEN repeat every 1-2 weeks Podophyllotoxin 0.5% gel bid x 3days,then 4 days off — to be repeated for 4 weeks. Prophylaxis for HPV (for Cervieul CA & warts) ~ Inj Gardasil IM 0,2 and 6 months. Therapeutic Guidelines | Obstetrics & Gynecology 39 GENERAL INSTRUCTIONS for all sexually transmitted infections: © Treat all parmers © Avoid sexual intercourse tll treatment completion. © Barrier contraception/ educate about sa © Rescreening in 3 months. sex practices, * DOXYCYCLINE: Drug induced PHOTOSENSITIVITY, use sun sercen ACYCLOVIR: headache, GI upset, impaired renal function, tremors, agitation, lethargy, confusion, coma 2. Urinary Tract Infection Uncomplicated: Tab Bactrim DS PO bid x3 days or ‘Tab Nitrofurantoin 100mg PO gid x Sdays. (with food) In pregnancy: Treat asymptomatic UTI ‘Tab Amoxicillin 250mg PO tid or "Tab Macrobid 100mg PO hid x 10 days. Pyclonephritis: Acute Uncomplicated: "Tab Ciprofloxacin 500mg PO bid x 10 days or ‘Tab Augmentin 625mg PO bid x 14 days. Inpatient: Inj Ceftriaxone 1g TV bid for 48 hours then switch to oral drugs + Inj Gentamicin 50mg IV q8h for 24 hours. 3. Vulvovaginitis a. Candidiasis: ib Miconazole 200mg PV qhs x3 days or ‘Tab Nystatin (100,00 unit) vaginal tah PV qhs x 14 days or ‘Tab Fluconazole 150mg PO stat dose. Prophylaxis: 4 or more infection per year ~"Tab Fluconazole 150mg PO every 3days for 3 doses. Maintenance:'Tab Fluconazole 150mg PO each week. Monitor liver enzymes every 1-2 months, b. Bacterial vaginosis: “Tab Flagyl S00mg PO bid x 7days.(with food) ©. Trichomonas vaginalis: b Flagyl 2g PO for 1 dose. or ‘Tab Flagyl 500mg PO bid x Zdays.(with food), treat partner. d. Atrophic vaginitis: ‘Topical Estrogen eream 0,5 to 2g daily to be applied locally. 40 NAC OSCE | A Comprehensive Review 4, Pelvic Inflammatory Disease (PID) a. Outpatient: Inj Ceftriaxone 250mg IM stat dose + Tub Doxycycline 100mg PO bid x 14days. b. Inpatient: Inj Cefoxitin 2g IV qh + Inj Doxycycline 100mg IV q12h. Continue IV for 48 hrs & then tab Doxycycline 100mg PO bid x 14 days. Reportable disease, treat partners, rescreening after 4-6 weeks incase of documented infection. 5. Dysfunctional Uterine Bleeding (DUB) a. Mild DUB: + NSATDs—Tab Mefenamie acid 500mg PO tid x 5 days, + Anitfibrinolytics ~Tranexamic acid 500mg PO tid x 5 days, Combined OCPS + Mirena / Provera + Tab Progestin one tab OD in first 10-14days, . Severe DUB: + Inj Premarin 25mg IV q4h +'Tab Gravol 50mg PO 4h, + With Tab Ovral PO tid till bleeding stops (24hrs), THEN bid for 2 days, THEN od for Says. + Continue conventional OCPs if pregnancy not desired 6.Dysmenorthea “Tab Ibuprofen 400mg PO gid from 1st day of menstrual cycle. + Oral Contraceptive Pills, + Important to rule ont secondary causes of dysmenorrhea. 7. Endometriosis a) NSAIDs : Tab Ibuprofen 400 ng PO gid till symptoms last. b) Oral Contraceptive pills ©) Tab Provera 10-20 mg PO OD. Tab Danazol 600-800 mg PO OD for 6 months. ©) GnRH Agonist: Inj Leuprolide 3.75 mg IM once a month for 6 months. Inj Goserelin 3.6 mg SC every 28 days for 6 months. Use GnRH Agonist along with Estrogen/Progesterone add back therapy. (To reduce the side cffcets of bone Loss.) ‘Therapeutic Guidelines | Obstetrics & Gynecology 41 8. Hormone Replacement Therapy (HRT) 2) Only Estrogen - Tab Premarin 0.625mg PO OD ( only estrogen) b) Cyelie Dose -Tab Premarin 0.625mg PO OD and Tab Provera 510mg PO OD from days 1-14. ©) Standard dose - Tab Prempro (premarin 0.625mg and provera 2.5mg) combination pill PO OD. 4) Pulsatile ~Tab Premarin 0.625mg PO OD and Low dose Tab Provera 15 mg PO OD. Given as 3 days on and 3 days off. ¢) Transdermal : Estradiol transdermal patch twice daily and Tab Provera 2.5 mg PO OD. 9. Emergency contraception OTC no prescription needed. Take within 72 hours of unprotected intercourse. Tab Ovral 2 tabs PO qi2h x 2 doses (has Levonorgestrel 0.Smy/dose + estrogen 0.1mg/dose) + Tab Benadryl 10mg 1 hr before dose (emesis induced by Estrogen). Plan B (Tab Levonorgestzel 0.7Smg/tab) one tab q12hes x 2 doses. 10. Group B Streptococcus (GBS) in pregnancy + Inj Penicillin G 5 MU IV then 2.5 MU IV q4h till delivery. + Penicillin allergic: Inj Cefazolin 2.g IV then 1g q8h or Inj Clindamycin 900 mg IV q8h or Inj Erythromycin 500 mg TV q6h. 11. Pregnancy Induced Hypertension (PIH) a Initial: To maintain DBP<100 . abetalol 20mg IV bolus every 10-20 mins pen. (C/I asthma,CHF) + Tab Nifedipine XL 10mg PO very 20-30 mins prn Inj Hydralazine Smg or 10mg IM every 20 mins prn, then 5 or 10mg every 3 hrs pen (S/W: fetal tachycardia, maternal headache, palpitations) b. Maintenan + Tab Methyldopa 250-500mg PO bid — qid. ‘Tub Metoprolol 25-100mg PO bid. + Tab Labetalol 100-400my PO qid. Anticonvulsant therapy: Inj Magnesium sulfate 4g IV bolus over 20 min, followed by maintenance of 2-4 g/hout. Monitor signs of Magnesium toxicity ~ depressed deep tendon seflexes, decreased respiratory reflex, anutic, hypotonic, CNS or cardiac depressio Antagonist to Magnesium sulfate: Calcium gluconate(10%) 10 ml TV over 2 minutes. 4. Avoid these antihypertensives: + ACE(-) & ARBs— neonatal renal failure, teratogenic, IUGR. + Atenolol -TUGR + Thiazide diuretics ~ maternal fluid depletion, 42 NAC OSCE | A Comprehensive Review 12. Ectopic Pregnancy a) Inj Methotrexate 50mg/m’ BSA IM stat dose. b) Repeat beta hCG levels weekly till <1. ©) Contraception till beta heg returns to SmIU/ml or less. d) Do CBC, LETS 15. Hyperemesis Gravidarum ‘Tab Diclectin (10 mg Doxylamine with 10 mg Pyridoxin) started as 1 tab GAM + 1 tab qPM + 2 tabs qhs. Maximum 8 tabs per day. 14, Drugs contraindicated in pregnancy + Chloramphenicol: Gray baby syndrome + Erythromycin: Maternal liver damage (used only if allergic to penicillin) + Fluoroquinolone: Cartilage damage. + Metronidaz: + Sulfa drugs: Miscarriage in 1© trimester and kernicterus in 3" trimester Anti-metabolite, high risk in 1° trimester and breast feeding. Can lead to miscarriage. +” Tetracyclines: Staining of teeth in children. + ACE inhibitors: IUGR, oligohydranminos, fetal renal defects. * Anticonvulsants: ’) Phenytoin: Fetal hydantoin syndrome ~ IUGR, facial dysmorphogenesis, cardiovascular defects, congenital anomalies of hand & foot, umbilical hernia and congenital anomalies, ii) Valproic acid: Lumbosacral spina bifida with meningomyelocele or meningocele, often accompanied by midfacial hypoplasia, deficient orbital ridge, prominent forehead, congenital heart disease, and decreased postnatal growth. iii) Carbamazepine: Unique facial appearance and underdevelopment of the fingers, toes, and nails; developmental delay. iv) Phenobarbital: Cleft palate/lip, congenital heart disease, intra-cranial hemorrhage. aginal adenosis, adenocarcinoma, uterine malformations in female fetuses. ’s cardiac anomaly, goiter, hyponatremia. Congenital facial paralysis with or without limb defects (Mobius syndrome) and Neural tube defects + Retinoids: Deformities of the cranium, ears, face, imbs, and liver, hydrocephalus, microcephaly, heart defects, cognitive defects, craniofacial alteration, cleft palate, neural tube defects, cardiovascular malformations and kidney alterations. Therapeutic Guidelines | Obstetrics & Gynecology 43 Warfarin: High risk of spontaneous abortion, stillbirehs, IUGR. Fetal Warfarin Syndrome: Deformities of the axial and appendicular skeleton, hypoplastic nose, optic atrophy, mental retardation, brachydactyly, scoliosis, mental retardation, intra~ cranial hemorrhage. Alcohol: High incidence of abortion and still births, IUGR. Fetal Alcohol Syndrome: Decreased muscle tone and coordination, cognitive impairmen ASD/VSD, narrow small eyes with large epicanthal folds, small head, small mid-face, indistinct philtrum, thin upper lip. igarette smoking: IUGR, placental abruptio/ prev Cocaine: IUGR, microcephaly, prematurity, mental retardation, . spontaneous abortion Therapeutic Guidelines | Pediatrics 45 Pediatrics 1 Acute Bronehi a. Mild distress: oral/IV hydration, antipyretics for fever, humidified O,, VENTOLIN 0.03ce in 3m! NS by face mask q20min and then qlhr. b. Moderate to severe distress: all the above + Ribavirin in high risk groups like congenital lung cliscase, congenital heart disease, bronchopulmonary dysplasia, immunodeticient patients. Antibiotics, ipratropium, systemic corticosteroids have no use. 2. Acute Oni Media (AOM) a) First line: Tab Amoxicillin 80-90mg/ky/day PO divided qh for 10d. If allergic ~’Tab Azithsomycin 10mg/kg/day OD for 3 days.’To be given if child > 6months old. b) Second line: Tab Augmentin 90mg/kg/day divided q12h for 10 days or Tab Cefuroxime 30mg/kp/day divided bid for 10 days, Avoid FLUQROQUINOLONES under 16 years age. 3. Asthma + Classification (NIH recommendations) a) Intermittent Asthma- Occasional exacerbations (Less than twice per week). b) Mild Persistent Asthma- Frequent exacerbations (>twice weekly, but not daily). ©) Moderate Persistent Asthma- Daily symptoms ly Beta Agonist use d) Severe Persistent Asthma~ Continuous Symptoms and frequent exacerbations. + Acute Management i. O, (to maintain O, saturation > 9006). fi, Fluids, if dchydrated. fii, B, Agonist : Salbutamol (Ventolin)- 0.03 ce/kg in 3ce NS every 20 minutes for 3 doses then 0.15-0.3 mg/kg (not to exceed 10 mg) every 1-4 hours as needed or 0.5 mg/kg/hour by continuous nebulization. un bromide (Atrovent) 1ce added to each of first 3 salbutamol masks. one 2mg/kg in ER, then Img/kg PO OD x 4d. svere— Ipratroy vy. Steroids: Inj Predni 4, Bacterial Tracheitis * Airway management, keep child calm. + Humidified 0, + Nebulized racemic epinephrine(1:1000 solution) in 3ml NS, 1-3 doses, ql-2h. + Inj Ceftriaxone 75-100mg/kg/day q24hrs + Inj Vancomycin 40mg/ke/day in divided doses every 6-8h. 46 NAC OSCE | A Comprehensive Review 5. Bacterial Pneumonia Newborn (under 3 weeks old) 1, Admit all newborns with Pneumonia, 2. Antibiotic regimen (Use 2-3 antibiotics combined) a) Antibiotic 1: Ampicillin i. Age <7 days + Weight <2 kg: 50-100 mg/kg divided q12h. + Weight >2 kg: 75-150 mg/kg divided q8b. ii, Age >7 days + Weight <1.2 kg: 50-100 mg/kg divided q12h + Weight 1.2-2 kg: 75-150 mg/kg divided qBh. + Weight >2 kg: 100-200 mg/kg divided qoh. Gentamicin (dosing below if >37 weeks old) + Age <7 days: 2.5 mg/kg repeated qi2b. + Age >7 days: 2.5 mg/kg repeated qBh. 3: Cefirtaxime (optional) + Age <7 days: 100 mg/kg divided q12h. + Age >7 days: 150 mg/kg divided q8h. 3. Organisms requiring additional antibiotic coverage i. Methicillin Resistant Staphylococcus Aureus (MRSA}-Vancomycin a) Age <7 days © Weight < 1.2 kg: 15 mg/kg IV OD. + Weight 1.2—2 ky : 10-15 my/kg IV q12-18h, + Weight > 2 kg 10-15 mg/kg IV q8-12h. b) Age > Zdays , weight > 2 kg : 45-60 mg/kg/day in divided IV qh. ii, Chlamydia trachomatis-Exythromycin 30-50 mg/kg/d PO divided q8h. b) Antibiotic ©) Antibion Management: Age 3 weeks to 3 months 1, Outpatient (if afebrile without respiratory distress) i. Azithromycin 10 mg/kg day 1, 5 mg/kg days 2-5 PO. ii, Erythromycin 30-40 mg/kg/day PO divided qoh x1 0days. 2, Inpatient {if febrile or hypoxic) i, Laj Erythromycin 40 mg/kg/day IV divided q6h and ii, One of the following antibiotics if febrile: + Inj Ceforaxime 200 mg/kg/day TV divided qh. + Inj Cefuroxime 150 me/ky/day IV divided qh. 3. Critically il i, Inj Cefotaxime as above and Inj Cloxacillin or ii, Inj Cefuroxime alone as above Therapeutic Guidelines | Pediatrics 47 Management: Age 3 months to 5 years 1. Outpatient (if afebrile without respiratory distress) a) Co nitial parenteral antibiotic at diagnosis: + Inj Ceftriaxone 50 mg/kg/day up to 1 gram IM x1 dose. + Start oral antibiotics concurrently as below. b) First-line oral agents: + Amoxicillin 90 mg/kg/day PO divided qh x7-10d. ¢) Alternative oral agents: + Amoxicillin-Clavulanie Acid (Augmentin) . + Erythromycin. + Clarithromycin. Azithromycin. 2. Inpatient (af febrile or hypoxic): a) Inj Cefotaxime 150 mg/kg/day IV divided qoh. b) Inj Cefuroxime 150 mg/kg/day IV divided qh. ©) Ifeonfirmed Pneumococcal Pneumonia: + Inj Ampicillin alone 200 mg/kg/day LV divided qh. 3. Critically il: a) Option 1 + Inj Cefotaxime 150 mg/kg/day IV divided q6h and + Inj Erythromycin 40 mg/kg/day IV divided qoh. b) Option 2 + Inj Cefuroxime 150 mg/kg/day IV divided 8h and + Inj Cloxacillin 150-200 mg/kg/day IV divided gob, Management: Age 5 to 18 years 1. Outpatient: a) First-line oral agents: i. Erythromycin 40 mg/kg/day PO divided qoh x 7-104. ii, Clarithromycin 15 mg/kg/day PO divided q12h x 7-10d. iii, Azithromycin + Day 1:10 mg/kg day 1 PO (maximum 500 mg). + Days 2-5: 5 mg/kg/day PO (maximum 250 mg). b) Preumococeal Pneumonia confirmed: i, Amoxicillin 90 mg/kg/day PO divided qh x 7-104. 2. Inpatient: a) First line and in critical illness: i. Inj Cefuroxime 150 mg/kg/day IV divided q8b and ii, Inj Erythromycin 40 mg/kg/day IV divided géh. b) Paeumococeal Pneumonia confirmed: i, Inj Ampicillin 200 mg/ky/day IV divided gh. 48 NAC OSCE | A Comprehensive Review 6. Croup (Laryngotracheobronchitis) 2) Humidified O, b) _ Nebulized racemic epinephrine(1:1000 solution) in 3m NS, 1-3 doses, ql-2h : + Child < 6mths: 0.25inl + Child > mths: 0.5m + Adolescent: 0.75ml ©) Dexamethasone 0.6mg/kg IM/IV/PO, max dose 10mg, given as a single dose. 7. Bpiglottitis + Suspect epiglotttis if child has fever, ill looking, dyspnea, dysphonia, loss of voice, stridor, sudden in onset. + Investigations: Pharyngeal swab and culture Blood culture Lateral X-ray neck (Thumbprint sign) ABG,CBC Endoscopy in ER + Treatment: Intubation IV fluid IV Cefuroxime 8. Streptococcal Pharyngitis (Group A Streptococcus) Melsaate Criteria ~ no cough, tender anterior cervical lymph nodes, erythematous tonsils with exudate, fever > 38°C, age 3-14 years. a) If 1 symptom only — no culture or antibiotics needed. b)_ If> 1 symptom, culture positive ~ treat with antibiotics: Penicillin V 40 mg/kg/day PO divided bid x 10 days. Erythromycin 40mg/kg/day PO divided tid x 10 days. ‘Acetaminophen for fever or pain. ©) Invasive GAS: needs admission — Inj Clindamycin 40 mg/kg divided into 3-4 doses and Inj Penicillin 250 000 ~ 400 000 Ukg/day divided into 6 doses x 10 days. 9. Whooping Cough (Pertussis) a) Exythromycin 40-5Omg/kg/day PO divided gid x 10d. b) Azithromycin 10mg/ky/PO OD day! Smg/ky PO OD day2 to days. (preferred) ©) Isolate for 5 days of treatment, d) Erythromycin to all the houschold members. Therapeutic Guidelines | Pediatrics 49 10. Bacterial Meningitis (Reportable disease) a) Inj Dexamethasone 0.6 mg/kg/day IV in 4 divided doses. Start within 1 hour of 1° antibiotic dose. b) Ampicillin: i, Agee 1 month ~ 50 mg/kg IV q8-12h. ii, Age>1 month ~ 50 mg/kg IV qoh. ©) Ceftaxime: i. Age < L month ~ 50 mg/kg TV g8-12h i, Age>1 month ~ 200 mg/kg/day IV divided q6-8h. d) Ceftriaxone: i, Ages 1 month — 50-75 mg/kg IV divided q12-24h Age> 1 month 100 mg/kg/d IV divided q12h ©) Gentamycin: 2-2.5 mg/kg TV qBh. f) Vancomyein: 15 mg/kg qéh 1V x 7-14 days. g) Prophylaxis for contacts i. He Influenzae: Rifampin 20 mg/kg/day up to 4 days. ii, N. Meningitides : + Rifunpin + Children: 10 mg/kg PO q12h x 2 days (max 600 mg), + Adults: 600 mg PO q12h x2 days. + Ciprofloxacin (adults) 500mg PO for one dose. + Ceftriaxone : + Age<15 years: 125 mg IM for one dose. + Ages 15 years: 250 mg IM for one dose. 11. Febrile Seizures In ER :Tnj Diazepam 0.2 —0.5mg/kg TV glSmins till b) Home : Diazepam rectal suppository. ©) Investigate & teat the cause of fever 12, Urinary Tract Infection (UTD) Oral Treatment- for 7-14 days. i, Cofixime (Suprax) 8 mg/kg PO divided bid or ii, Cefpodoxime (Vantin) 10 mg/kg PO divided bid or iii, Cefprozil (Cefzil) 30 mg/kg PO divided bid or ix. Cephalexin (Keflex) 50-100 mg/kg PO divided qi. 2. IV antibiotics Inj Cefotaxime 50-150mg/ke/day divided q4-6h or ‘75me/kg/day divided q12-24h. Tnj Ceftriaxone 3. UTI Prophylaxis i, Bactrim (mg TMP/10mg SMZ per kg) at bedtime qhs. 50 NAC OSCE | A Comprehensive Review aa a) General Measures: i. ABC management. ii, Oxygen. iii, IVE iy, Nebulised beta-agonist (Albuterol). bb) Anaphylaxis with airway compromise: Epinephrine (1:1000 solution) 0.01ml/ky SC/TM(upto 0.3m) ©) Urticaria, Pruritus of Flashing: Inj Diphenhydramine 25-50mg IM/V every 6hrs prn. Orally same dose qéh x 3days. @) Prevention: i, Medical alert bracelet. ii, Strict avoidance of allergen. i. BpiPen. iv, Allergy testing and desensitization therapy. 14, Anemia in + 6mg/kg/day elemental iron bid-tid. + Investigate the cause of anemia, 15. Dose of Tylenol in children 4) 40-60 mg/kg/day PO divided qéhr prn (not to exceed 5 doses/24 hours). b) Neonates: 10-15 mg/kg PO q6-8hr pn, 16, Immunization Schedule for Infants and Children in Canada 52 NAC OSCE | A Comprehensive Review Psychiatry 1. Agitation & Aggression Agitation can be defined as excessive verbal and/or motor behavior. It can readily escalate to aggression, which can be cither verbal (vicious cursing and threats) or physical (toward objeets or people). Goals of treatment (Nonpharmacological treatment) Create a safe environment for treatment Decrease stimulation Permit patient to ventilate his or her feelings, but this may need to be cut short if the degree of agitation is escalating and there is clear danger to self and others. Behavioral approaches include never turning your back to an agitated patient, talking softly rather than shouting, and inquiring about what specific needs the patient may have Prevent further episodes of agitation or aggression ‘Delirium, First-generation Antipsychotics (FGA) Haloperidol ; 0.5-2.5 mg POM bid ANTIPSYCHOTICS S/E S Delirium or acute confusional state is a common and severe neuropsychiatric syndrome with core features of acute onset and fluctuating course, attentional deficits and generalized severe disorganization of behavior. Treatment of delirium requires treatment of the underlying causes. Antipsychoties are first-line treatment. Haloperidol is the most effective medication for decreasing agitation in delirious patients. First generation antipsychotic Loxapine and second generation (atypical) antipsychotics such as Olanzapine,Risperidone and Quetiapine can also be used. Benzodiazepines should be reserved for cases of alcohol withdrawal. + Loxapine : 12.5-50 mg/day PO soa Parcreond | Second generation (atypical) antipsychotics (SGA) reameiptaa ee: 4 + Olanzapine : 5-10 mg/day PO, 2.5-10 mg TM (repeat 2h and 6h prn | Syndrome (NMS) to max of 30 mg/24 h) Sedation + Risperidone : 0.5-2 mg/day PO port st elaeaall + Quetiapine : 25-100 mg/day PO Hypefprolactinemia and Benzodiazepines I dysfunction. + Lorazepam : 0.5-1 mg POM q6-8h i Oxazepam : 10-15 mg PO tid Therapeutic Guidelines | Psychiatry 53 Mania + Mania is a state of abnormally elevated or irritable mood, arousal, and/ or energy levels. Treatment of ‘mania involves both acute control of severe agitation by a mood stabilizer and long term mood stabilizers, Initially atypical antipeychoties such as Risperidone, Olanzapine or Quetiapine are effective, First-generation Antipsychotics + Haloperidol : 5-10 mg/day POM. Atypical antipsychotics + Risperidos mg/day PO + Olanzapine : 5-20 mg/day PO, 2.5-10 mg IM (repeat 2h and 6h prn to max of 30 mg/24 h) + Quetiapine : start with 100 mg/day PO; increase by 100 mg/clay as necded to 300-600 mg/day ided BID 2. Anxiety Disorders + Ansivty disorders are a group of conditions with exaggerated ansiousness and worry about a number of concerns persists for an extended period of time. Goals of treatment (Nonpharmacologic treatment) + Stress reduction and relaxation techniques such as meditation and low impact yoga is often helpful. + Cognitive behavioral therapy (CBT) + Reduction of consumption of caffeine and other stimulants. + Minimize use of alcohol Panic disorder + Panic attack or panic disorder involves sudden anxiety that occurs without warning, Symptoms can include chest pain, heart palpitations, sweating, shortness of breath, feeling of unreality, trembling, dizziness, nausea, hot flashes or chills, a feeling of losing contol, or a fear of dying, Panic attacks are extremely common - 10% to 2046 of the population experience a panic attack at some point in theit life. Some people start to avoid situations that might trigger a panic attack; this is called panic attack with agoraphobia. Panic disorder refers to recurring feelings of terror and eas, which come on unpredictably without any clear trigger. + SSRIs and SNRIs are the first choice in the treatment of panic disorders, Sclective serotonin reuptake inhibitors (SSRIs) ike Citalopram, Escitalopram, Fluoxetine, Paroxetine and Sertraline are all effective in reducing punie attacks, Serotonin norepinephrine reuptake inhibitor (SNRIs) eg. Venlafaxine is also used in panie disorder. + ‘There is a delay in the onset of response to these drugs which may be accompanied by initial agitation. Combining SSRI or SNRI with a brief course of low dose benzodiazepine ean increase adherence to medication and produce rapid response. + Other medication include Tricyclic antidepressants (TCAs) cg. Imipramine, Desipramine and Clomipramine and Monoamine oxidase inhibitors (MAOIs) eg. Phenelzine, 'Tranyleypromine. 54 NAC OSCE | A Comprehensive Review Selective serotonin reuptake inhibitors (SSRIs) + Paroxetine : 20-60 mg/day PO SEROTONIN SYN + Fluoxetine : 20-80 mg/day PO oO Nr: + Sertraline : 50-200 mg/day PO inus tachycardia, HTN, + Citalopram : 20-60 mg/day PO at at tyres: + Escitalopram : 10-20 mg/day PO flewa, muscle rigidity, Seats 5 taxi, ncoordnation, Serotonin norepinephrine seuptake inhibitor (SNRIs) PTI berapd ober + Venlafaxine :37.5-22.5 mg/day Ca eae ‘Tricyclic antidepressants (TCAs) Metiioeroide Feperolol + Clomipramine : 75-225 mg/day : + Desipramine : 75-300 mg/day ia eee + Imipramine : 75-300 mg/day Benzodiazepines * Alprazolam : 0.25 mg tid-qid, up to 1 mg PO qid + Clonazepam : 0.25-0.5 mg PO bid Monoamine oxidase inhibitors (MAOIs) + Phenelzine : 45-90 mg/day + Tranyleypromine : 20-60 mg/day Social Phobia involves excessive anxiety in social situations where people ‘ocial anxiety inelude small group anxiety include jons that eanse + Social anxiety, also known as social phobi fear being embarrassed or made fin of. Situations that can trigge discussions, dating, going to a party, and playing sports. Common symptoms of socia blushing, sweating, and dry mouth. People with social phobia often avoid social situ: anxiety. + SSRI and SNRI are mainstay drugs for the treatment for social phobia. Escitalopram, Fluvoxamine, Paroxetine, Sertraline and Venlafaxine may be used for milder cases, + Simple stage fright or fear of public speaking may respond to low dose Propranolol 10mg taken 30 minutes before the event. General Anxiety Disorder + Generalized anxiety disorder (GAD) is associated with continual excessive anxiety and worry about & number of things (c.g., work, money, children, and health). There is no specific source of fear. Symptoms can include muscle tension, trembling, shortness of breath, fast heartbeat, dizziness, dry mouth, nausea, sleeping problems, and poor concentration, CBT is the most effective psychosocial treatment but often takes 20 or mor sions to be effective. + SSRIs and SNRIs have become established as first line treatments for GAD. Bupropion and. Pregabalin are further choices, Law dose benzodiazepines can be used but dependence is « problem: Buspizone has « low abuse potential and is less sedating than benzodiazepines. + Buspirone : Sing hid-tid, up to 60 mg/day Therapeutic Guidelines | Psychiatry 55 + Pregabalin : Initial 150 mg/day in 2-3 divided doses, may be increased to 150 mg bid after 1 week if necessary Bupropion (Wellbutrin, Zyban): Use : Smoking cessation, second line Antidepressant. Antidepressant: Start 100 mg bid x 4 days > 100 m g tid. Obsessive-compulsive disorder (OCD) OCD involves recurring unpleasant thoughts (obsessions) and/or repetitive behaviours (compulsions). ‘The thoughts may be connected to the repetitive behaviours, For example, people who fear getting an infection may constantly wash their hands. Ac times, however, therc’s no connection at all between the thoughts and the behaviours. + CBT is the psychotherapy of choi SSRIs : Fluoxetine, Fluvoxamine, Paroxetine and Sertraline, in the usual antidepressant dosing range are the drugs of choice in Canada. It may take 6-8 weeks for symptoms to improve. Second line drugs include Clomipramine, Venlafaxine, Citalopram and Mirtazapine. Post-traumatic stress disorder (PTSD) + PTSD is associated with extreme anxiety that appears after a traumatic experience. Symptoms usually start within 3 months of the traumatic event but may take years to start, PTSD can be associated with sleep problems, nightmares, irritability, and anger. Feelings of guilt and unworthiness are common with PTSD. Traumatic experiences that can trigger PTSD include wars, plane crashes, natural disasters (cg. hurricane, carthquake), and violent crimes (e.g., rape, abuse), + SSRI and SNRI antidepressants have been shown to be effective in reducing the symptoms of PTSD. Fluoxetine, Paroxetine, Sertraline and Venlafuxine are first line options. 3. Dementia Dementia is a serious loss of cognitive ability in a previously unimpaired Zope person, beyond what might be expected from normal aging. Dementia is | ADAMADL Dressing | nota single disease, but rather a non-specific illness syndrome in which Eating affected areas of cognition may be memory, attention, language, and ‘Ambulation problem solving, It is normally required to be present for at least 6 months pene to be diagnosed, + The most common causes of dementia are Alzheimer's disease and vasculir | shopping dementia, It afiects about 1% of people aged 60-64 years and as many as | Housekeeping. 30-50% of people older than 85 years, ‘Accounting + Benzodiazepines must be used cautiously in the elderly patients due to | Food Making increase risk of falls and impaired cognition. [ieeporeaesy 56 NAC OSCE | A Comprehensive Review 4, Depression Nonpharmacological treatment + Cognitive behavioral and interpersonal psychotherapy are as effective as antidepressants in mild to moderate depression. Pharmacological treatment + Take medication daily, antidepressant must be taken for 2 to 4 weeks for effect to be noticeable. Medication must he taken even if patient is feeling better. SSRI + Paroxetine (Paxil): Start 20 mg ghs, increase 10mg every 2wks, max 60mg per day. + Bhuoxetine (Prozac): Start 20mg PO qd, avoid increasing more often than monthly, max 80mg PO per day + Sertraline (Zoloft): Start SOmg PO qAM, increase 50mg every 2 weeks, max 200mg per day + Fluvoxamine (Luvox): Start 25mg PO ghs x 3 days -> 50mg PO ghs x 7 days -> titrate 150-250 mg daily divided doses bid. + Citalopram (Celexa): Start 20mg PO gd, max 60mg, + Escitalopram (Lexapro): Start 10mg PO gd Monoamine Oxidase Inhibitor (MAO inhibitor): Use in Atypical depression, Refractory depression, + Isocarboxazid (Marplan) ~ 10 mg PO bid, max 60 mg per day: + Phenelzine (Nardil) ~ 15 mg PO tid, max 90 mg per day. + Tranyleypromine (Parnate) ~ 10-40 mg per day in divided doses, max 60 mg per day. Complication: Hypertensive crisis, Serotonin syndrome, Interaction with tyramine containing foods to bbe avoided strietly. Serotonin Norepinephrine Reuptake Inhibitors(SNRI) not used these days + Tricyclic Antidepressants: Amitriptyline 25 mg qhs, Nortriptyline (Pamelor) S/E: Anti-cholinergic ~ dry mouth, constipation, blurred vision, Anti-histaminergic — sedation, weight gain; Scrotonengie ~ sexual dysfunction; Orthostatic hypotension; Sinus tachycardia, SVT, Ventricular tachycardia, Prolonged QT interval, heart block; Withdrawal symptoms. Other : Venlafaxine (Effexor) 37.5 mg PO od 6. Psychosis In acutely psychotic individuals, short-acting parenteral antipsychotics either alone or in combination with a parenteral benzodiazepine may be recommended. Liquid formulations of atypical antipsychotics may be used as an alternative to intramuscular injections, Risperidone and Olanzapine are examples. Atypical anipaychotics: Clozapine ~ 12.5 mg PO qd or bid, titrate slowly upwards in increments of 25-50 mg/day “Target dose 1300 ~ 450 mg/day, max 900 mg/day. S/E: Agranulocytosis, Diabetes mellitus, hypertriglyceridemia, NOT 1” LINE Anti-psychotic, Order weekly blood counts for1 month and then q2 weeks, Therapeutic Guidelines | Psychiatry 57 + Olanzapine (Zyprexa) ~ Start 5-10 mg PO qd, inezease in 5 mg increments weekly as tolerated, max 20 mg/day. ld sedation, insomnia, dizziness, early AST & ALT Life- threatening neurological coe peers with misde elevation, restlessness, weight yain, increased risk of diabetes jidity, fi mic mellitus and hyperlipides tabby, delta. + Quetiapine (Seroquel) ~ Start 25 mg PO bid tid, increase in 25- Elevated creatine 50 mg/day increments, target 300-400 mg/day divided doses bid EPCSPHOMInase tid, max 750 mg/day, | S/E: Headache, sedation, dizziness, constipation + Risperidone (Risperdal) ~ Start 1 mg PO bid, slow titration 2-4 mg PO daily or divided doses bid, max 16 mg/day. S/R: Insomnia, agitation, EPS, headache, ansiety, hyperprolactenemia, postural hypotension, constipation, dizziness, weight gain, ‘Typical antipsychotics: + Haloperidot (Haldol)- 5-10 mg PO, IM, IV. May repeat q30-60mins, max 300 mg per day. + Fluphenazine (prolixin) ~ 2.5 mg PO bid, max 40 mg per day. f - EXTRA PYRAMIDAL SIDE EFFECTS: Akinesia treat with Benztropine mg PO /IM/IV OD Dystonic reaction — treat with Lorazepam or benztropine. Dyskinesia Akathesia ~ creat with Lorazepam, Propranolol or Diphenhydramine. Perioral tremor Neuroleptic malignant Syndrome — Muscle rigidity, tremor, delirium, high fever, diaphoresis, hypertension, Discontinue drug. Give symptomatic treatment and supportive care.”Treatment with Dantrolene or bromocriptine. Tardive dyskinesia — Blinking, lip smacking, sucking, chewing, grimaces, choreoathetoid movements, tonic contractions of neck / back. ‘Treatment — Clozapine. 7. Mood stabilizers : Used in Bipolar disorder. + Lithium bicarbonate ~ Start 300 mg PO bid, then ine Serum levels — 0.6 — 1.2 mEq/l, monitor RF'Ts, TF TS. S/E: tremor, polydipsia, hypothyroidism, weight gain, nausea/voniting, diarrhea, ataxia, slurred speech, lack of coordination ‘Treatment of Lithium toxicity: Discontinue lithium. Check serum lithium levels, BUN, electrolytes, IV fluids ~ Normal saline. Hemodialysis in case Li > 2 mmol/L, coma, shock, severe dehydration, deterioration, unresponsive to treatment. 40 900 ~ 1800 mg per day divided doses. 58 NAC OSCE | A Comprehensive Review + Divalproex (Epival) 300-1600 mg/day PO bid. Monitor LPTs weekly x 1 month, then monthly. S/E: Liver dysfunction, nausca/vomiting, diarrhea, ataxia, drowsiness, tremor, sedation, hair loss, weight gain thrombocytopenia, neural tube defects in carly pregnancy. + Carbamazepine (Tegretol) 750- 3000 mg/day PO tid. Monitor weekly CBC due to risk of agranulocytosis. S/E; Nausea/vomiting, hepatic toxicity, ataxia, drowsiness, confusion, nystagmus, diplopia, transient leukopenia, agranulocytosis, drug reaction (SS), neural tube defect in early pregnancy. 8. Medications causing sexual dysfunction 1. Antiandrogen Medications + Spironolactone + Ketoconazole + Finasteride 2. Antihypertensives + Centrally acting sympatholyties (e.g. Clonidine) + Peripherally acting sympatholytics (c.g. Guanadrel) + Beta Blockers + Thiazide Diuretics 3, Antidepressant Medications lective Serotonin Reuptake Inhibitors (SSRI) + Tricyclic Antidepressants * MAO inhibitors 4, Sedative-Iypnotic Medications + Barbiturates, + Benzodiazepines 5. Drug Abuse + Alcohol Abuse + Hero + M: + Methadone + Tobaceo abuse 6, Other Medications + Anticholinergic Medications + Antipsychotic Medications + 12 Receptor Blockers Therapeutic Guidelines | Psychiatry 59 9. Substance abuse + Alcohol withdrawal: + Tab Diazepam 20 mg PO q1-2h pr. + Observe for 1-2 hours and re-assess. + Inj Thiamine 100 mg IM then 100 mg PO OD x 3 days. + Maintain hydration, + feral Diazepam not well tolerated then switch to Inj Diazepam 2-5 mg IV/min— maximum 10-20 mg qth, or S/L Lora + Ifsevere liver dysfimnction severe asthma, respiratory failure or age> 65 years present ~ Lorazepam PO/SL/IM 1-4 mg q 1-2h. + Hallucination present - Haloperidol 2-5 mg IM/PO q1-4h ~ max 5 doses/day along with Diazepam 20 mg x 3 doses as seizure prophylaxis. + Wernicke's syndrome: Thiamine 100 mg PO OD x 1-2 weeks. + Korsakoft’s syndrome: Thiamine 100 mg PO bid/tid x 3-12 months. + Opioid Intoxication: + ABCs + IV Glucose * Inj Naloxone (Narcan) 0.4 mg — 2mg IV. + Intubation and mechanical ventilation may be required for decreased level consciousness. + Cocaine Overdose: + ABCs + Inj Diazepam 2-5 mg IV/min ~ maximum 10-20 mg q1h ( to control seizures). + Propranolol or labetalol to treat hypertension and arrhythmia. + Hallucinogens: LSD, mescaline, psilocybin, MDMA. + Symptomatic treatment and supportive care. + Decreased stimulation * Benzodiazepines or antipsychotics might be required. + Pheneyclidine: + Room with minimal stimulation, + Inj Diazepam IV for muscle spasms or seizures. + Haloperidol wo suppress psychotic behavior Clinical Examination ‘This i a blank page Clinical Examination 63 Abdominal Examination 1, Steps before beginning examination + Introduce yourself “Lam De.___, your attending physician and I'll be examining you today. At any point of the examination you feel uncomfortable, please let me know and I'll stop the examination right there.” + Wash/Sanitize hands + Explain to the patient what you are about to do and gain informed consent. + Look for medical equipment/therapies (e.g. drains, colostomy/ileostomy bags). + Verbalize the steps of the examination and your findings. + Use proper draping techniques. 2. Inspection + General inspection of the patient : Is patient comfortable at rest? Do they appear to be tachypnocic? + Examine the patient's hands for presence of koilonhychia (iron deficiency), leukonychia (hypoalbuminemia), clubbing (TBD, cocliac disease, cirrhosis), palmar erythema, tar staining or Dupuytren’s contracture. + Ask the patient to hold their hands out in font of them looking for a any tremor and then get them to extend their wrists up towards the ceiling keeping the fingers extended and look for flapping (asterixis in hepatic encephalopathy). + Examine the face, check the conjunctiva for pallor. Also check the sclera for c. Look at the buccal mucosa for any obvious ulcers which could be a sign of Crohn's disease, B12 or iron deficiency. Also look at the tongue. [fit is red and fat it could be another sign of anaemia, as could angular stomatitis. Check state of dentition — pigmentation of oral mucosa (Peutz-Jegher's syndrome), telangectasia, candidiasis, + Examine the neck for an enlarged left supraclavicular lymph node, A palpable enlarged supraclavicular (Virchow's) node is known as Troisier's Sign, may be a sign of malignancy. Virchow's node drains the thoracic duet and receives lymphatic drainage from the entire abdomen as well as the left thorax. ‘Therefore, enlargement of this node may suggest metastatic deposits from a malignancy in any of these areas, + Examine the chest, in particular look for gynaecot nacvi.’These are both stigma of liver pathology. + Inspect the abdomen and comment on any obvious abnormalities such as scars, masses and pulsations. Also note if there is any abdominal distension/ascitcs. Look for distended veins, striae, Cullen's/Grey- ‘Turner's signs (pancreatitis), Sister Mary Joseph's nodule (widespread abdominal cancer) wundi nce of S or more spider tia in men and the pres 64 NAC OSCE | A Comprehensive Review 3, Auscultation: + Listen with the diaphragm next to the umbilicus for up to 30 seconds. + Listen for bowel sounds ~ absent (e.g. Ileus, peritonitis) tinkling (bowel obstruction) 4, Palpation + Palpation of the abdomen should be performed in a systematic way using the 9 named segments of the abdomen: right and left hypochondrium, right and left flank, right and left iliac fossa, the umbilical area, the hypochondrium and the suprapubic region. + Tfapatient has pain in one particular area you should start as far from that area as possible. The tender area should be examined last as they may start guarding making the examination very difficult. + Initial examination should be superficial using one ally, you should examine deeper. This is performed with two hands, one on top of the other. + Feel for organomegaly, particularly of the liver, spleen and kidneys. Pulpation for the liver and spleen is similar, both starting in the right iliac fossa. For the liver, press upwards towards the right hypochondrium. You should try to time the palpation with the patient's breathing as this presses down on the liver. Ifthe liver is distended, its distance from the costal margin should be noted. + Palpating for the spleen is as for the liver but in the direction of the left hypochondrium. The edge of the spleen which may be felt if distended is more nodular than the liver. + To feel for the kidneys you should place one hand under the patient in the flank region and the other hand on top. You should then try to ballot the kidney between the two hands. hand.Once you have examined all 9 areas sup. 5. Percussion Percussion over the abdomen is usually resonant, over a distended liver it will be dull. Percussion can also be used to check for ‘shifting dullness'- a sign of ascites. With the patient lying flat, start percussing from the midline away from you, If the percussion note changes, hold you finger in that position and ask the patient to roll towards you, Again percuss over this area and if the note has changed then it suggests presence of fluid such as in ascites, [tis also appropriate at this time to check for pedal edema. 6. You should mention to the examiner at this point that you would like to finish the examination with an ion of the hernial orifices, the external genitalia and also a rectal examination Clinical Examination 65 Cardiovascular Examination 1. Steps before beginning examination your attending physician and I'll be examining you today. At cl uncomfortable, please let me know and I'll stop the examination + Inuoduce yourself: “I am Ds. any point of the examination you fi right there.” + Wash/Sanitize hands + Explain to the patient what you are about to do and gain informed consent. + Look for medical equipment/therapies (e.g: GIN spray, ECG pads, oxygen) + Verbalize the steps of the examination and your findings 2. Inspection + Start by observing the patient fiom the end of the bed. You should note whether the patient looks comfortable. Are they cyanosed or flushed? + Respiratory rate, chythm and effort of breathing, + Chest shape, chest movements with respration (symmetrical/assymetrical), skin (scars/nevi) + Inspect the nails for clubbing, splinter hemorrhages (infective endocarditis), koilonychia (iron deficiency anemia). + Inspect fingers for capillary refill time, peripheral cyanosis, osler's nodes (infective endocarditis) and nicotine staining, + Inspect palms for palmar erythema, Janeway lesions and xanthomas. + Take the radial pulse, assess the rate and rhythm.A¢ this point you should also check for a collapsing pulse —a sign of aortic incompetence. Locate the radial pulse and place your palm over it, then raise the arm above the patient’s head. A collapsing pulse will present as a knocking on your palm. At this point you should say to the examiner that you would like to take the blood pressure.’They will usually tell you not to and give you the value. + Inspect the sclera for any signs of jaundice, anaemia and corneal arcus. You should also look for any evidence of xanthelasma. + Whilst looking ar the face, check for any malar facies, look in the mouth for any signs of anaemia such as glossitis, check the colour of the tongue for any cyanosis, and around the mouth for any angular stomatitis — another sign of anaemia + Assess jugular venous pressure (JVP), ask patient to turn their head to look away from you. Look across the neck between the two heads of sternocleidomastoid for a pulsation then measure the JVP. + Examine the chest, or praecordium for any obvious pulsations, abnormalities or scars, remembering to check the axillae as well. 66 NAC OSCE | A Comprehensive Review 3. Palpation + Palpate praecordium trying to locate the apex beat and describe its location anatomically: The normal location is in the Sth intercostals space in the mid-clavicular line, + Palpate for any heaves or theills. A thrill isa palpable murmur whereas a heave is a sign of left ventricular hypertrophy. Feel for thes over the praccordium, 4, Auscultation + Mitral valve ~ where the apex beat was felt. + Tricuspid valve —on the left edge of the sternum in the 4th intercostal space. + Pulmonary valve ~ on the left edge of the sternum in the | 2nd intercostal space. + Aortic valve ~ om the right edge of the sternum in the 2nd intercostal space. How many heart sounds are heard? Are the heart sounds normal in character? Any abnormal heart sounds? If you hear any abnormal sounds you should describe them by when they occur and the type of sound they are producing, Are there any murmurs? Can you hear any rub? Peeling the radial pulse at the same time can give good indication as to when the sound occurs — the pulse occurs at systole. Furthermore, if you suspect a murmur, check if it radiates. Mitral murmurs typically radiate to the left axilla whereas aortic murmurs arc heard over the left carotid artery. + To further check for mitral stenosis you can lay the patient on their left side, ask them to breathe in, then out and hold it out and listen over the apex and axilla with the bell of the stethoscope. + Aortic incompetence can be assessed in a similar way but ask the patient to sit forwad, repeat the breathe in, out and hold exercise and listen over the aortic area with the diaphragm. 5. With patient sitting up percuss back for pleural effusion (cardiac failure) 6. Finally assess for any pedal & sacral oedema 7. Finish by thanking the patient and ensuring they are comfortable and well covered. Clinical Examination 67 Peripheral Vascular Examination 1. Steps before beginning examination + Introduce yourself :“Lam Dr, , your attending physician and I'l be examining you today. At any point of the examination you feel uncomfortable, please let me know and I'll stop the examination right there.” + Wash/Sanitize hands + Explain to the patient what you are about to do and gain informed consent. + Look for medical equipment/therapies (eg. GIN spray, ECG pads, oxygen) + Verbalize the steps of the examination and your findings. 2. Inspection + General observation of the patient, arms from the finger tips to the shoulder and legs from the groin and buttocks to the tocs. Comment on the general appearance of the arms and legs, siz, swelling, symmetry, skin colos, hair, scars, pigmentation including any obvious muscle wasting, Note colour and texture of nails. + Any signs of gangrene or pre-gangrene such as missing toes or blackening of the extremities. + "The presence of any ulcers ensure you check all around the fect including behind the ankle,’These may be venous of arterial ~ one defining factor is that venous ulcers tend to be painless whereas arterial are painful. + Any skin changes such as pallor, change in colour (eg purple/black from haemostasis or brown from haemosiderin deposition), varicose eczema or sites of previous ulcers, atrophic changes and hair loss. + Presence of any varicose veins ~ often seen best with the patient standing, 3. Palpation + Assess the skin temperature. Starting distally, feel with the back of your hand and compare each limb to the other noting any difference. + Check capillary return by compressing the nail bed and then releasing it. Normal colour should return within 2 seconds, If this is abnormal, perform Buerger’s Test. This involves raising the patient’s feet to 45°. In the presence of poor arterial supply, pallor rapidly develops. Following this, place the feet over the side of the bed, eyanosis may then develop. + Any varicosities which you noted in the observation should now be palpated. If these are hard to the touch, or painful when touched, it may suggest thrombophlebitis. + Palpate peripheral pulses.’These are: Carotid — only palpate one curotid ata time Radial —use the pad of three fingers Brachial — may use thumb to palpate Femoral — fecl over the medial aspect of the inguinal ligament. 68 NAC OSCE | A Comprehensive Review Popliteal ~ ask the patient to flex their knee to roughly 60° keeping their foot on the bed, place both hands on the front of the knee and place your fingers in the popliteal space. Posterior tibial ~ felt posterior to the medial malleolus of the tibia. Dorsalis pedis ~fecl on the dorsum of the foot, lateral to the extensor tendon of the great toe. You should compare these on both sides and comment on their strength. * Check for radio-femoral delay, Palpate both the radial and femoral pulses on one side of the body. The pulsation should occur at the same time. Any delay may suggest coarctation of the aorta. 4, Auscultation : listen for femoral and abdominal aortic bruits 5. Special Tests + Allen’Test : Ask the patient to make a tight fist and elevate the hand. Occlude the radial and ulnar arteries with firm pressure.’The hand is then opened. It should appear blanched (pallor ean be observed at the finger nails). Release either the Ulnar or radial artery pressure and the color should return in 7 seconds. If the palm does not redden immediately, this suggests arterial insufficiency. + Straight Leg Raise and Refill Test (Buerger's’Test) : Raise the leg 45° to 60° for 30 seconds until pallor of the feet develops and observe empty veins. Sit the patient upright and observe the feet. In normal patients, the feet quickly turn pink (within 10-15 seconds). If, pallor persists for more than 10- 15s or there is development of a dusky cyanosis (rubor), this suggests of arterial insufficiency. + Test for incompetent Saphenous Vein : Ask the patient to stand and note the dilated vaticose veins. ‘Compress the vein proximally with one hand and place the other hand 10-15 cm distally. Briskly compress and decompress the distal site. Normally, the hand at the proximal site should fecl no impulse, however with varicose veins a transmitted pulse may be felt + Trendelenburg Mancuver (Retrograde filling) : Ask the patient to lie down, Elevate the leg, and empty the veins by massaging distal to proximal, Using a tourniquet, occlude the superfic the upper thigh. Ask the patient to stand. Ifthe tourniquet prevents the veins from re-filling rapidly, the site of the incompetent valve must be above this level ive, xt the sapheno-femoral junction. If the veins re-fill, the communication must be lower down. Observing the same protocol, proceed down the leg until the tourniquet controls re-filling. As necessary, test: tbove the knee = to assess the mid-thigh perforator + below the knee ~ to assess competence between the short saphenous vein and popliteal vein If re-filling cannot be controlled, the communication is probably by one or more distal perforating veins in veins. Clinical Examination 69 Respiratory Examination 1. Steps before beginning examination * Introduce yourself :“Lam Dr.___, your attending physician and I'll be examining you today. At any point of the examination you feel uncomfortable, please let me know and I'll stop the examination «Sane and + Explain to the patient what you are about to do and gain informed consent. + Look for medical equipment/therapies (e.g. inhalers, oxygen). + Verbalize the steps of the examination and your findings. 2. Inspection * — General look of the patient. Check whether they are comfortable at rest, is patient tachypnoeic? Are they using accessory muscles? Are there any obvious abnormalities of the chest? Check for any clues around the bed such as inhalers, oxygen masks or cigarcttes. + Inspect the hands, hot, pink peripheries may be a sign of carbon dioxide retention. Look for any signs of clubbing, cyanosis, hypertrophic pulmonary ostcourthropathy, dupytren's contacture and nicotine staining, Assess for carbon dioxide retention flap/salbutamol tremor. + Take the patient’s pulse, Afier you have taken the pulse itis advisable to keep your hands in the same position and subtly count the patient's respiration rate. + Laspoct the face, ask the patient to stick out their tongue and note its colour — checking for cyanosis. ~ Horner's sydrome (Pancoast tumour) , plethora (polycythemia). + Look for any use of accessory muscles such as the sternocleidomastoid muscle. Also palpate for the left supraclavicular node (Virchow’s Node) as an enlarged node (Troisier's Sign) may suggest metastatic lung cancer. ‘+ Examine the chest and back. Observe the chest for any deformities (barrel chest, kyphoscoliosis, pectus excavarum, pectus carinarum), symmetry of expansion, dilated veins, intercostal recession, 3. Palpation + Palpate the chest. Feel between the heads of the two clavicles for the trachea, sce if itis deviated + Feel for chest expansion, Place your hands firmly on the chest wall with your thumbs meeting in the midline. Ask the patient to t deep breath in and note the distance your thumbs move apart. Normally this should be at least 5 centimetres. Measure this at the top and bottom of the lungs as well ason the back. 70 NAC OSCE | A Comprehensive Review 4. Percussion. Percussion should be performed on both sides, comparing similar arcas on both sides. Start by tapping on the clavicle which gives an indication of the resonance in the apex. Then percuss normally for the entire lung fields, Hlyper-resonance may suggest a collapsed lung where as hypo-resonance or dullness suggests consolidation such as in infection or a tumour, Be sure to perform this on the back as well 5. Vocal Fremitus Check for tactile vocal fremitus, Place the medial edge of your hand on the chest and ask the patient 10 say “99, Do this with your hand in the upper, middle and lower arcas of both lungs. 6. Auscultation Do this in all areas of both lungs and on front and back comparing the sides to each other. Listen for any reduced breath sounds, or added sounds such as crackles, wheezes or rhonchi Bronchial sounds ++/-egophony at edge Bronchial Occasional crackles Away Hyper-resonant Absent Nil Towards lesion Dull Decreased Late inspiratory severe crackles Central Resonant (normal)| Normal Decreased if ‘ish by examining the lymph nodes in the head and neck. Start under the chin with the submental nodes, move along to the submandibular then to the back of the head at the occipital nodes. Next palpate the pre and post auricular nodes. Move down the cervical chain and onto the supraclavicular nodes, Clinical Examination n Central Nervous System Examination 1. Steps before beginning examination + Introduce yourself : “I am Dr. _ your attending physician and I'll be examining you today. At any point of the examination you feel uncomfortable, please let me know and I'll stop the examination right there.” + Wash/Sanitize hands + Explain to the patient what you are about to do and gain informed consent. + Look for medical equipment/therapies (e.g, walking aids). + Verbalize the steps of the examination and your findings. 2. Cranial Nerve Examination 1) The Olfactory nerve (CN I) is simply tested by offering something familiar for the patient to smell and identify — for example coffee or vinegar. 2) The Optic nerve (CN ID) is tested in five ways: + The acuity is casily tested with Snellen charts. This should be assessed both with the patient wearing any glasses or contact lenses they usually wear and without them, * Colour vision is tested using Ishara plates, these identify patients who are colour blind. + Visual fields are wsted by asking the patient to look directly at you and wiggling one of your fingers in each of the four quadrants. Ask the patient to identify which finger is moving, Visual inattention can be tested by moving both fingers at the same time and checking the patient identifies this, + Visual reflexes comprise direct and concentric reflexes. Place one hand vertically along the nose to block any light from entering the eye not being tested. Shine a pen torch into one eye and check that the pupils on both sides constrict, This should be tested on both sides + Finally fundoscopy should be performed on both eyes. 3) Eye movements: Oculomotor nerve (III), Trochlear nerve (IV) and Abducent nerve (VI) are involved in movements of the eye. Asking the patient to keep their head perfectly still direetly in front of you, you should draw two large joining H's in front of them using your finger and ask them to follow your finger with their eyes. It is important the patient does not move their head. Always ask if the patient experiences any double vision and if so when is it worse. Also look for ptosis and assess saccadic eye movements, 4) The Trigeminal nerve (CN V) is involved in sensory supply to the face and motor supply to the muscles of mastication, Initially test the sensory branches by lightly touching the face with a piece of cotton wool and then with a blunt pin in three places on each side ~ around the jawline, on the cheek and on the forehead. The corneal reflex should also be examined as the sensory supply to the comea is from this nerve. This is done by lightly touching the cornea with the cotton wool, This should cause the patient to shut their eyelids. For the motor supply, ask the patient to clench their teeth together, observing and feeling the bulk of the masseter and temporalis muscles. Then ask them to open their mouth against resistance, Finally perform the jaw jerk on the patient by placing your left index finger on their chin and striking it with a tendon hammer. This should canse slight protrusion of the jave. 72 NAC OSCE | A Comprehensive Review 3. 5) ‘The Facial nerve (CN VI) supplies motor branches to the muscles of facial expression. Therefore, this nerve is tested by asking the patient to ercase up their forchead (raise their eyebrows), close their eyes and keep them closed against resistance, puff out their cheeks and show you their teeth. 6) The Vestibulocochlear nerve (CN VIII) provides innervation to the hearing apparatus of the ear and ‘can be used to differentiate conductive and sensori-neural hearing loss using the Rinne and Weber tests, For the Rinne rest, place a sounding tuning fork on the patient’s mastoid process and then next to their ear and ask which is louder, a normal patient will find the second position louder. For Weber's test, place the tuning fork base down in the centre of the patient's forehead and ask if itis louder in cither ear. Normally it should be heard equally in both eats. 7). The Glossopharyngeal nerve (CN IX) provides sensory supply to the palate. It can be tested with the ‘gag reflex or by touching the arches of the pharynx. 8) The Vagus nerve (CN X) provides motor supply to the pharynx. Asking the patient to speak gives a good indication to the efficacy of the muscles. You should also observe the uvula before and during, the patient saying ‘nah’. Check that it lies centrally and does not deviate on movement. 9) The Accessory nerve (CN XI) gives motor supply to the sternocleidomastoid and trapezius muscles. “To test it, ask the patient to shrug their shoulders and turn their head against resistance. 10) ‘The Hypoglossal nerve (CN XII) provides motor supply to the muscles of the tongue, Observe the tongue for any signs of wasting or fasciculations. Then ask the patient to stick their tongue out. If the tongue deviates to cither side, it suggests a weakening of the muscles on that side. Serebellar Examination Gait: + Ask the patient to stand up. Observe the patient's postut + Ask the patient to walk, e.g. to the other side of the root walking aid, allow them to do s0, + Observe the different gait components (heel strike, toe liftoff). Is the gait shuffling / waddling / scissoring / swinging? + Observe the patients arm swing and take note how the patient turns around as this involves good blance and co-ordination. + Ask the patient to walk heel-to-toc to assess balance. + Perform Romberg’s test by asking the patient to stand unaided with his eyes closed. [F the patient sways or loses balance this test is positive. Stand near the patient in case he falls, and whether they are steady on their feet. dd back. Ifthe patient normally uses a Co-ordination: * Look for a resting tremor in the hands. + ‘Test tone in the arms (shoulder, elbow, wrist) + Test for dysdiadochokinesis by showing the patient to clap by alternating the palmar and dorsal surfaces of the hand. Ask to do this as fast as possible and repeat the test with the other hand. + Perform the finger-to-nose test by placing your index finger about two feet from the patients face. Ask him to touch the tip of his nose with his index finger then the tip of your finger. Ask him to do this as fast as possible while you slowly move your finger. Repeat the test with the other hand. + Perform the heel-to-shin test. Have the patient lying down for this and get him to run the heel of one foot down the shin of the other leg and then to bring the heel back up to the knee and start again. Repeat the test with the other leg. Clinical Examination Pe Upper Limb Neurological Examination 1, Steps before beginning examination + Introduce yourself “Lam Dr. , your attending physician and I'l be examining you today. At any point of the examination you feel uncomfortable, please let me know and I'll stop the examination sight there.” + Wash/Sanitize hands + Explain to the patient what you are about to do and gain informed consent. + Use proper draping techniques, verbalize the steps of the examination and your findings. 2. Inspection + General inspection of patient: general comfort, abnormal posture/movements, muscle wasting. + The upper body should be exposed for this examination. Observe the patient's arms, look for any muscle wasting, fasciculations or asymmetry. + Examine the tone of the muscles. Start proximally at the shoulder, feeling how easy the joint is to move passively. Then move down to the elbow, wrist and hand joints again assessing each one's tone in turn, + Assess for spastic catch, clasp-knife rigidity, led-pipe or cog-wheel rigidity 4. Power + Next assess the power of each of the muscle groups. — Shoulder abduction (C5) & Shoulder adduction (C5/C6/C7) = Elbow flexion (C5/C6) & Elbow extension (C7) = Wrist flexion (C8) & Wrist extension (C8) = Finger flexion (C8), Finger abduction (1'1), Finger adduction (11) = Thumb abduction (C8) 5. Reflexes + ‘There are three reflexes in the upper limb - the biceps, triceps and supinator reflexes + The biceps reflex (C5/C6) is wsted by supporting the patient's arm, with it flexed at roughly 60°, placing your thumb over the biceps tendon and hitting your thumb with the tendon hammer. It is, vital to get your patient to relax as much as possible and for you to take the entire weight of their arm. + The triceps reflex (C6/C7) is elicited by resting the patient's arm across their chest and hitting the triceps tendon just proximal to the elbow. * Finally, with their arm rested on their abdomen, locate the supinator tendon (C5/C6) as it crosses the radius, place three fingers on it and hit the fingers.’This should give the supinator reflex. If you struggle with any of these reflexcs, asking the patient to clench their teeth should exaggerate the reflex. 74 NAC OSCE | A Comprehensive Review 6. Sensation ‘This is tested in a number of ways. You should test light touch, pin prick, vibration and joint position, sense and proprioception, [Ask the patient to place their arms by their sides with their palms facing forwards. Lightly touch the patient's sternum with a piece of cotton wool so that they know how it feels. Then, with the patient's eyes shut, lightly touch their arm with the cotton wool. The places to touch them should test each of the dermatomes. Tell the patient to say yes every time they feel the cotton ‘wool as it felt before. Then repeat this using a light pin prick. "To assess vibration you should use a sounding tuning fork. Place the fork on the patient's stermum to show them how it should feel. Then place it on the bony prominence at the base of their thumb and ask them if it feels the same. If it does, there is no need to check any higher, If it feels different yon should move to the radial stylus and then to the olecranon until it feels normal, Finally, proprioception. Hold the distal phalanx of the thumb on either side so that you ean flex the interphalangeal joint. Show the patient that when you hold the joint extended, that represents "Up" whereas when you hold it flexed that represents ‘Down’. Ask the patient to close their eyes and, having moved the joint a few times hold it in one position - up or down. Ask the patient which position the joint is in. 7. Coordination Pronator drift — Ask patient to extend arms in front of them in supination and to close their eyes. A positive result eccurs when the arm falls downwards and pronates (cerebral damage), in cerebellar lesions the arms may rise ‘Assess for dysdiadochokinesia Assess for finger to nose coordination and intentional tremor. Function is a very important part of any neurological examination as this is the area which will affect people's day to day lives the most. For upper limb you should ask people to touch their head with both hands and then ask them to pick up a small object such as a coin which each hand Finish by thanking the patient and ensuring they are comfortable and well covered. Clinical Examination 75 Lower Limb Neurological Examination 1. Steps before beginning examination + Introduce yourself :Tam Dr. , your attending physician and I'll be examining you today. Ar any point of the examination you feel uncomfortable, please let me know and I'll stop the examination right there.” + Wash/Sanitize hands + Explain to the patient what you are about to do and gain informed consent. + Verbalize the steps of the examination and your findings. + Make sure patient is adequately exposed, use proper draping techniques 2. Inspection + Observe the patient's legs, look for any muscle wasting, fasciculations or asymmetry. 3. Tone + Start by examining the tone of the muscles. Roll the leg on the bed to see if it moves casily and pull up on the knee to check its tone, Also check for ankle clonus by placing the patients leg turned outwards on the bed, moving the ankle joint a few times to relax it and then sharply dorsiflexing it. Any further movement of the joint may suggest clonus. 4, Power + Next assess the power of each of the muscle groups. = Hip flexion (1.1/1.2) & Hip extension (L5/$1) — Hip abduction (L2/L3) & Hip adduetion (L2/L3) = Knee flexion (L5/S1) & Knee extension (L3/L4) — Ankle dorsiflexion (L4/L5) & Ankle pkantar flexion (S1/S2) Big toe flexion (S1/S2) 5. Reflexes + Test the patient's reflexes.'There are three reflexes in the lower limb - the knee reflex, the ankle jerk and the plantar reflex - elicited by stroking up the lateral aspect of the plantar surface. + The knee reflex (1.3/1.4) is tested by placing the patient's leg flexed at roughly 60°, taking the entire weight of their leg with your arm and hitting the patellar tendon with the tendon hammer. I is vital to get your patient to relax as much as possible and for you to take the entire weight of their leg. + The ankle jerk (S1/S2) is clicited by resting the patient's leg on the bed with their hip laterally rorated. Pull the foot into dorsiflexion and hie the calcancal tendon, + Finally, with their leg out straight and resting on the bed, run the end of the handle of the tendon hammer along the outside of the foot. This gives the plantar reflex (S1). An abnormal reflex would see the great toe extending. If you struggle with any of these reflexes, asking the patient to clench their teeth should exaggerate the reflex. 76 NAC OSCE | A Comprehensive Review 6. Sensation + ‘The final testis sensation. However, this is tested in a number) of ways, You should test light touch, pin prick, vibration and { vi = joint position sense and proprioception. _—— »\ + ‘Ask the patient to place their legs out straight on the bed. )% Lightly touch the patient's sternum with a piece of cotton wool |» i so that they know how it feels. Then, with the patient's eyes 2 | shut, lightly touch their leg with the cotton wool.’The places to a \ touch them should test each of the dermatomes = make sure | 7 | | you know these! Tell the patient to say yes every time they feel | o\ \ the cotton wool as it felt before. Then repeat this using alight [cj .y pin prick + To-assess vibration you should use a sounding tuning fork. “a \ Place the fork on the patient's stemum to show them how it |, ° ! should feel. Then place it on their medial malleolus and ask a te i\ them if it feels the same. If it does, there is no need to check ‘ Ls any higher, If it feels different you should move to the tibial he epicondyle and then to the greater trochanter until it feels (i) \ normal. A \ + Finally, proprioception, Hold the distal phalanx of the great toe kaverime, 5p on either side so that you can flex the interphalangeal joint. Anterior Posterior Show the patient that when you hold the joint extended, that represents ‘Up! whereas when you hold it flexed that represents ‘Down’, Ask the patient to close their eyes and, having moved the joint a few times hold it in one position ~ up or down. Ask the patient which position the joint isin. 7. Function is avery important part of any neurological examination as this is the area which will affect people's day to day lives the most. For the lower limb you should assess the patient's walking. Observe their gait and check for any abnormalities. Whilst they are standing you should perform Romberg's test. Ask the patient to stand with their feet apart and then close their eyes. Stand next to the patient in case he falls. Any swaying may be suggestive of a posterior column pathology. 8 Finish by thanking the patient and ensuring they are comfortable and well covered. mination _ Fi Musculo-skeletal system : Spine/Back 1. Steps before beginning examination + Introduce yourself:"I am Dr.___ yon attending physician and I'll be examining you today. At any point of the examination you feel uncomfortable, please ler me know and I'll stop the examination right then + Wash/Sanitize hands + Explain to the patient what you are about to do and gain informed consent. + Ensure patient is adequately exposed. + Look for medical equipment/therapies + Show empathy. + Verhalize the steps of the examination and your findings. 2. Inspection + Ask for patient's vitals + Observe patient : Is patient sitting comfortably? Gait? Position of comfort. + Observe the patient from behind : — Pelvic and shoulder symmetry, palpate the pelvic brim to check for symmetry. = Scoliosis — Gibbus (dorsal spines abnormally prominent) + Observe patient from side = Kyphosis = _ Increased lumbar lordosis + Check the spine for SEADS : S: Swelling, E: Erythema, bulk), D: Deformity, $: Skin changes/scars/bruising fhymosis, A: Atrophy/asymmetty (muscle 3. Range of Motion + Flexion : In the standing position by asking the patient to touch the toes. Normal - 90° ‘The normal spine should lengthen more than 5 cm in the thoracic area and more than 7.5 em in the lumbar area on forward flexion. + — Extension : Stabilize the patient, ask the patient to bend backwards. Normal — 30° + Lateral flexion : ask the patient to slide their hand straight down the thigh, first on the right and then on the left, keeping the hips straight. + Observe for restricted movement and loss of symmetry. + Test for facet joint disease : Ask patient to extend their back as far as possible and to rotare (pain suggests facet joint pathology). 4, Palpation + Examine the back and palpate for areas of muscle spasm and tenderness (paraspinal muscles). + Palpate spinous processes with thumb for tenderness + Sacroiliac joints, sacro iliac dimples, ask for tenderness, 78 NAC OSCE | A Comprehensive Review 5. Ankylosing spondylitis tests Chest expansion : Measuse with a tape measure (should be >Sem) Schober's Test : Draw a horizontal line 10em above and one Sem below the dimples of Venus (the distance between these lines should increase to >20em during lumbar flexion ~ in ankylosing spondylitis the distance will not increase to >20em) Distance of tragus to wall when patient is standing with their back to the wall (useful for monitoring). 6. Cervical and thoracic movements (patient sitting on edge of bed) Cervieal movements = Flexion (ask patient to touch chin to chest) = Extension (ask patient to look to the ceiling as far back as possible) — Lateral flexion (ask patient to touch their ear to the shoulder keeping the shoulder still) — Spurling Maneuver : Extend head back & bring ear towards shoulder. Give gentle axial pressure on the bead. If patient complains of pain radiating from head to ipsilateral arm ~ diagnosis of Radiculopathy is made. = Rotation (ask patient to look over the left and right shoulder) = Perform these movements passively if active movements are restricted. ‘Thoracic rotation :ask patient to fold their arms and twist around, 7.'Tests with patient lying on their back Straight leg raising test : ask the patient to lie with the spine on the table and to relax completely. With the knee fully extended, first one leg and then the other is slowly lifted and flexed at the hip. This produces stretch on the sciatic nerve, at which point sciatic pain is produced. If this maneuver produces pain in the hip or low back with radiation in the sciatic area, the test is considered positive for nerve root irritation. The angle of elevation of the leg from the table at the point where pain is produced should be recorded. FABER (Flexion Abduction External Rotation) : Ask the patient to lie supine on the exam table. Place the foot of the affected side on the opposite knee. Pain in the groin area indicates a problem with the hip and not the spine. Press down gently but firmly on the flexed knee and the opposite anterior superior iliac crest. Pain in the sacroiliac area indicates a problem with the sacroiliac joints, Bowstring test: Once the level of pain has been reached, flex the knee slightly and apply firm pressure with the thumb in the popliteal fossa over the stretched tibial nerve. Radiating pain and paraesthesiae ‘suggest nerve root irritation. 8.'Tests with patient lying on their abdomen Lasegue's sign: With the patient supine and hip flexed, dorsiflexion of the ankle causes pain or muscle spasm in the posterior thigh if there is lumbar root or sciatie nerve irritation. Femoral stretch test: With the patient prone and the anterior thigh fixed to the couch, flex each knee in turn. This causes pain in the appropriate distributions by stretching the femoral nerve roots in L2~ L4. The pain produced is normally aggravated by extension of the hip.'The test is positive if pain is, felt in the anterior compartment of thigh. Clinical Examination 79 Hip Examination 1, Steps before beginning examination Introduce yourself :“I am Dr. , your attending physician and I'll be examining you today. At any point of the examination you feel uncomfortable, please let me know and I'll stop the examination right there.” Wash/Sanitize hands Explain to the patient what you are about to do and gain informed consent. Ensure patient is adequately exposed Look for medical equipment/therapies Ask which hip is painful, show empathy. Verbalize the steps of the examination and your findings. 2. Inspection of hip (with patient standing up) 3. Gait — ask patient to walk across the floor. Look for any abnormalities, While the patient is standing, check the hip for SEADS :S: Swelling, E: Erythema, ecchymosis, A: Atrophy/asymmetry (muscle bulk), D: Deformity, $: Skin changes (erythema/scars/abscess/sinuscs) Leg length discrepancy Whilst the patient is still standing, perform the ‘Trendelenberg test. This is done by asking the patient to alternately stand on one leg. Stand behind the patient and feel the pelvis. It should remain at level or rise slightly. If che pelvis drops markedly on the side of the raised leg, then it suggests abductor muscle weakness on the leg the patient is standing on. hip, knec, foot movements, length of stride. it |Festinating ~ Parkinson's ds. High stepping ~ Polio, MS 3. Inspection & Palpation of hip (with patient lying down) Scissor ~ Spastic cerebral Inspection for hip and groin swellings (hernia, lymphadenopathy, ae TD iocwicl’s saphenous vatix, effusion) ataxia, tabes dorsalis Inspect for obvious fixed flexion ponerse sturge Palpate anterior hip for lumps and tenderness. : Palpate thi suggest trochanteric bursitis. ss which might 4, Leg-length difference Make an approximate judgment by aligning the medial malleoli and looking for discrepancy. Measure true and apparent leg-length if appropriate. Truc leg length disesepancy is found by measuring from the anterior superior iliae spine to the medial malleolus. Apparent leg length discrepancy is measured from the umbilicus to the medial malleolus. 80 NAC OSCE | A Comprehensive Review 5. Active and passive movements + Ascess active flexion, extension, abduction and adduction. = Flexion: Flex the knec to 90 degrees and passively flex the hip by pushing the knee towards the chest. = Extension : is performed by placing your hand under the patient's ankle and asking them to push your hand into the bed. + Passively assess internal and external rotation of the hip (with hips at 90° flexion) = Internal rotation : performed with the knee flexed and by everting the knee for internal rotation — External rotation : performed with the knee flexed and inverting it for external rotation. 6. Special tests + Thomas test : Plice your hand under the patient's lumbar spine to stop any lumbar movements and fully flex one of the hips. Observe the other hip, iit lifts off the couch then it suggests a fixed flexion deformity of that hip. + FABER (Flexion Abduction External Rotation) : Ask the patient to lie supine on the exam table. Place the foot of the affected side on the opposite knee, Pain in the groin area indicates a problem with the hip and not the spine, Press down gently but firmly on the flexed knee and the opposite anterior superior iliac crest. Pain in the sacroiliac area indicates a problem with the sacroiliac joints. Clinical Examination 81 Knee Examination 1. Steps before beginning examination Introduce yourself: “I am Dr. , your attending physician and I'll be examining you today. At any point of the examination you feel uncomfortable, please let me know and I'll stop the examination right there. ‘Wash/Sanitize hands Explain to the patient what you are about to do and gain informed consent. Ensure patient is adequately exposed (up to above knees). Look for medical equipment/therapies Aske about knee locking, giving way and pain, show empathy. Verbalize the steps of the examination and your findings. 2. Inspection Gait : Ask the patient to wall for you. Observe any limp or obvious deformities such as sears or muscle wasting, Check if the patient has varus (bow-legged) or valgus (knock-knees) deformity. Also observe from behind to see if there arc any obvious popliteal swellings such as a Baker's cyst. While the patient is lying on the bed, make a general observation. Look for symmetry, redness, muscle ‘wasting, scars, ashes or fixed flexion deformities. 3. Palpation Checle the temperature using the backs of your hand, comparing it with other parts of the leg, Palpate the border of the patella for any tenderness, behind the knee for any swellings, along all of the joint lines for tenderness and at the point of insertion of the patellar tendon. Finally tap the patella to see if there is any effusion deep to the patella, Landmarks of the knee : Tibial tuberosity, patellar tendon, quadriceps tendon, medial and lateral femoral condyles. Peripatellar arca : push patella medially and rub right underneath the medial facet of patella and look for tenderness ( Patellar — femoral stress 5°). Joint line tenderness : bend the knee 90°, palpate medial and lateral joint line, Patella apprehension test ~ Move patella around and observe patient's face for pain. 4, Range of Motion Active flexion and extension of knee — Observe for restricted movement and for displacement of patella. Passive flexion and extension of knee — feel for crepitus. Straight leg raise ~ assessment of extensor apparatus. 82 NAC OSCE | A Comprehensive Review 5. Special tests ‘Tests to ‘Tests to Tests to Detect Arthritis: Crepitas: Crepitus is the sensation that is fele when rough cartilage or exposed bone is rubbing as the knee is bent, The examiner will feel, and may heas, this grinding as the knee is bent back and forth. Deformity: As the knee cartilage is worn away, the kaees may become progressively knock-kneed or bow-legged. Limited Motion:’Vhe range of motion of the knee typically becomes limited if arthritis, bone spurs, and swelling prevents normal mobility. Detect a’Torn Meniscus: Joint Line Tenderness Joint line tenderness is a very non-specific test for a meniscus tear. The arva of the meniscus is felt, ‘and a positive testis considered when there is pain in this area McMurray’ Test McMhurray’s testis performed with the patient lying flat on his back and the examiner bending the knee. A click is felt over the meniscus tear as the knee is brought from full flexion to full extension. Detect an ACL Tear: Lachman Test ‘The Lachman test is the best test to diagnose an ACL. tear. With the knee slightly bent, the examiner stabilizes the thigh while pulling the shin forward, A tom ACL allows the shin to shift too far forwacd Anterior Drawer Test ‘This testis also performed with the patient lying flat on his back. The knee is bent 90 degrees and the shin is pulled forward to check the stability of the ACL. Detect Other Ligament Injuries: Posterior Drawer Test ‘The posterior drawer is performed similarly to the anterior drawer test. This test detects injury to the PCL. By pushing the shin backward, the function of the PCL is tested, Collateral Ligament Stability ide-to-side stability of the knee detects problems of the collateral ligaments, the MCL and LCL. With the patient lying flat, and the knee held slightly bent, the shin is shifted to each side. Damage to the LCL or MCL will allow the knee to "open up" excessively, a problem called varus (LCL) or valgus (MCL) instability. Detect Kneccap Problems: Patellar Grind “The patient lies supine with the leg extended. The examiner reproduces the patient's knee pain by pushing the kneceap down and asking the patient to flex his thigh museles. Damaged cartilage can cause a grinding sensation called crepitus Patellar Tenderness ‘The examiner can slightly lift up the kneecap and place direct pressure on the under surface of the kneecap. By doing so, the examiner is looking for sensitive regions of cartilage. Patellar Apprehension ‘This is a sign of an unstable kneecap. While the examiner places pressure on the kneecap, the patient may complain of the sensation that the kneecap is going to 'pop out’ of its groove, Clinical Examination 83 Foot and Ankle Examination 1, Steps before beginning examination + Introduce yourself :“1 am Dr. , your attending physician and I'll be examining you today. At any point of the examination you feel uncomfortable, please let me know and I'll stop the examination right there.” + Wash/Sanitize hands + Explain to the patient what you are about to do and gain informed consent. + Ensure patient is adequately exposed (up to above knees). * Look for medical equipment/therapies + Ask if patient is able to bear weight, show empathy. + Verbalize the steps of the examination and your findings. 2. Inspection + Gait : watch the patient walk, observing for a normal heel strike, toe-off gait. Also look at the alignment of the toes for any valgus or varus deformities. Assess ability to weight-bear on affected side. + While patient is standing check the foot arches checking. for pes cavus (high arches) or pes planus (Hat feet). + Inspection of the foot with patient sitting and feet overhanging Check the foot and ankle for SEADS : S: Swelling, E: Erythema, ecchymosis, A: Atrophy/asymmetry (muscle bulk), D: Deformity, $: Skin changes/scars/bruising. ~ Check the symmetry, nails (psoriasis), skin, oe alignment, look for toe cluwing, joint swelling and plantar and dorsal calluses. + Finally you should look at the patient's shoes, note any uneven wear on either sole and the presence of any insoles. 3. Palpation of ankle/foot + Feel each foot for temperature, comparing it to the temperature of the rest of the leg + Feel for distal pulses. = Squeeze over the metatarsophalangeal joints observing the patient's face for any pain, + Palpate over the midfoot, ankle and subtalar joint lines for any tendemess. Fee! the Achilles tendon for any thickening or swelling, Palpate medial and lateral malleoli for any tenderness. 84 NAC OSCE | A Comprehensive Review Range of Motion = Assess all active and passive movements of the foot. These movements are inversion, eversion, dorsiflexion and plantarflexion. = Subtalar joint inversion and eversion Ankle joint ~ dorsiflexion and plantar flexion = _ Big toc dorsiflexion and plantar flexion = Mid-tarsal joints - which are tested by fiaing the ankle with one foot and inverting and everting the forefoot with the other. 5. Special tests + Ankle Anterior Drawer Test - assesses for lateral ankle sprain Patient is seated, stabilize the tibia with one hand while grasping heel and pulling it anteriorly with the other. Greater than 3 mm anterior movement may be significant. 1 cm is significant and indicates anterior talofibular ligament rupture. Positive Test ~ laxity in the ligament with exaggerated anterior translation + Talar'Tile Test ~ assesses integrity of the deltoid ligament/lateral ankle sprain Patient is seated, stabilize the leg and foot while adducting and inverting the calcaneus apply a varus force. The calcaneus is then abducted and everted applying a valgus force. Positive Test - pain or laxity in the ligament + Thompson's Test ~ assesses for Achilles’ tendon rupture Patient is prone, squeeze the gastrocnemius and soleus muscles while not ankle and foot Positive Test - no movernent or plantarflexion at all indicates a 3rd degree strain of the Achilles’ tendon + Plantar Fasciitis Test ~ assesses for inflammation of the plantar fascia Patient is supine, dorsiflex the ankle and extends all toes then press in the medial border of the plantar fascia Positive Test ~ pain is consistent with plantar fase nggany movement at the Ottawa Ankle rules For taking ankle series x-rays(AP and lateral ankle) ‘+ X-ray if there is pain over the malleolar zone AND tenderness on palpation of the medial/lareral malleolar tip and posterior aspect of medial/lateral malleolus OR + Patient unable to bear weight immediately and in ER, For foot series (AP and Lateral foot) + X-ray if there is pain in midfoot zone AND bony tenderness over the navicular or base of 5" metatarsal OR + Unable to bear weight immediately and in ER Clinical Examination 85 Shoulder Examination 1, Steps before beginning examination + Introduce yourself “Tam Dr. , your attending physician and I'll be examining you today. At any point of the examination you feel uncomfortable, please let me know and I'l stop the examination right there.” + Wash/Sanitize hands + Explain to the patient what you are about to do and gain informed consent. + Look for medical equipment/therapies, ensure patient is adequately exposed. * Ask which shoulder is painful. Verbalize the steps of the examination and your findings. 2. Inspection + Start by exposing the joint and observe the shoulder joint looking from the back, side and front for any scars, deformities or muscle wasting (SEADS). Also compare both sides for symmetry. + With the patient standing, ask the patient to place their hands behind their head and behind their back and observe for and deformiti 3. Palpation + Feel over the joint and its surrounding areas for the temperature of the joint as raised temperature may suggest inflammation or infection in the joint. + Systematically feel along both sides of the bony shoulder girdle, Start at the sternoclavicular joint, work along the clavicle fo the acromioclavicular joint + Feel the acromion and then around the spine of the scapula + Feel the anterior and posterior joint lines of the glenohumeral joint and finally the muscles around the joint for any tenderness. 4, Range of Motion * The movements of the + Ask the pa (extension), + Bring theirarm out to the side and up above their head (abduction), When testing adduction perform the scarf rest (The scarf test is performed with the elbow flexed to 90 degrees, placing the patient's hand on their opposite shoulder and pushing back, again look for any discomfort. ) + Rotation = Internal rotation : Ask the patient to place hands in the small of their back, and slide them up the back as far as possible. — External rotation : Ask patient to rotate their arms outwards, keeping the elbows flexed and by the side of the body. + Once all of these movements have heen performed actively, you should perform them passively and feel for any crepitus whilst moving the joints sat should start being performed actively. jent to bring their arm forward (flexion), bend theis arm at the elbow and push backwards 86 NAC OSCE | A Comprehensive Review 5, Special Tests Tests for Rotator Cult i. Supraspinatus + Empty Can Test ( tests integrity of Supraspinatus) : The patient stands with arms extended at the elbows and abducted in the scapular plane and with thumbs pointed to the floor. The examiner applies downward pressure to the arms and the patient attempts to resist. Positive test : Pain, muscle weakness or both + Apley's Scratch Test Reach over shoulder to "scratch" between scapula, Measure to which vertebrae thumb can reach. ii, Infraspinatus + External Rotation Lag Sign : The elbow is passively flexed to 90 degrees, and the shoulder is held at 20 degrees abduction (in the scapular plane) and near maximal external rotation by the examiner. ‘The patient is then asked to actively maintain the position of extemal rotation in abduction as the exatniner releases the wrist while maintaining support of the limb at the elbow. The sign is positive when a lag, or angular drop occurs. ‘The magnitude of the lag is recorded to the nearest 5 degre iii, Subseapularis © Gerber Lift-Off Test: With the patient's hand on the small of the back, the arm is extended and internally rotated. The examiner then passively lifts the hand off the small of the back, placing the arm in maximal internal rotation, The examiner then releases the hand. Ifthe hand falls onto the back because the subscapularis is unable to maintain internal rotation, the test result is positive. Patients with subscapularis tears have an increase in passive external rotation and a weakened ability. to resist internal rotation. “Tests for Shoulder Instability + Apprehension Sign for Anterior Instability : The test is performed by abducting the shoulder to 90 degrees, and then slowly externally rotating the shoulder toward 90 degrees. ‘A patient with anterior inferior instability will usually become "apprehensive" either verbally or with distressing facial expressions. “Tests for Subacromial Impingement + Neer Impingement Sign : Place one hand on the posterior aspect of the scapula to stabilize the shoulder girdle, and, with the other hand, take the patient's internally rotated arm by the wrist, und place it in fall forward flexion.If there is impingement, the patient will report pain in the range of 70 degrees to 120 degrees of forward flexion as the rotator cuff comes into contact with the rigid coracoacromial arch. + Hawkins Impingement Sign : The examiner places the patient's arm in 90 degrees of forward flexion and forcefully internally rotates the arm, bringing the greater tuberosity in contact with the lateral. acromion, A positive result is indicated if pain is reproduced during the forced internal rotation. Tests for Long Head of the Biceps + Speed's Maneuver : The patient's elbow is flexed 20 degrees to 30 degrees with the forearm in supination and the arm in about 60 degrees of flexion. ‘The examiner resists forward flexion of the arm while palpating the patient's biceps tendon over the anterior aspect of the shoulder, + Yergason test : The patient's elbow is flexed to 90 degrees with the thumb up. Forearm is in neutral ‘The examiner grasps the wrist, resisting attempts by the patient to actively supinate the forearm and flex the elbow. Pain suggests biceps tendonitis. Clinical Examination 87 Elbow Examination 1, Steps before beginning examination + Introduce yourself :“I am Dr. , your attending physician and I'll be examining you today. At any point of the examination you feel uncomfortable, please let me know and I'l stop the examination right there.” + Wash/Sanitize hands + Explain co the patient what you are about to do and gain informed consent. + Look for medical equipment/therapies, ensure patient is adequately exposed + Vorbalize the steps of the examination and your findings. 2. Inspection + SEADS : S: Swelling, E: Erythema, ecchymosis, A: Atrophy/asymmetry (muscle bulk), D: Deformity, S: Skin changes/scars/bruising, Check for a fixed flexion deformity. Look at the underside of the elbows to check for any psoriatic plaques, cysts or rheumatoid nodules. 3. Palpation + Feel the elbow, assessing the joint temperature relative to the rest of the arm. + Palpate the olecranon process as well as the lateral and medial epicondyles for tenderness (medial for golfer's elbow and lateral for tennis elbow), and cubital fossa for tenderness. + Palpate joint line with elbow flexed to 90” for tenderness and swelling. 4, Range of Motion + The movements at the elbow joint are all fairly easy to describe and assess.’These are flexion, extension, pronation and supination, Once these have been assessed actively they should be checked passively checking for power and erepitus. + ‘Test for varus / valgus instability. 5. Neurological Examination of hand Motor Sensory ~ Median nerve (thumb abduction) ~ Median nerve (pulp of index finger) ~ Radial nerve (wrist extension) ~ Radial nerve (1" dorsal interosseous space) - Ulnar nerve (finger abduction) ~ Ulnar nerve (pulp of 5 finger) 6. Special Tests + Tennis Elbow :‘Tennis elbow localises pain over the lateral epicondyle, particularly on active extension of the wrist with the elbow bent. Golfer's Elbow : Golfer's elbow pain localises over the medial epicondyle and is made worse by flexing the wrist, 88 NAC OSCE | A Comprehensive Review Hand and Wrist Examination teps before beginning examination + Introduce yoursel any point of the exa right there.” + Wash/Sanitize hands + Explain to the patient what you are about to do and gain informed consent. + Look for medical equipment/therapies + Verbalize the steps of the examination and your findings. _,your attending physician and I'll be examining you today. At ou feel uncomfortable, please let me know and I'll stop the examination 2.Inspection + Inspect hands : — Skin (rashes, Gottron's patches, nodules, Raynaud's phenomenon, sclerodactyly, atrophy) — _ Nails (pitting, onycholysis, splinter haemorthages, clubbing) = Muscles (swelling, wasting) = Joints (swellings, subluxation / deviation of wrist, swan neck / Boutoniere’s deformity, Heberden's/Bouchard’s nodes, 7, deformity of thumb) — Inspect palm (palmar erythema, pallor, cyanosis), muscle wasting. + Inspect elbows Psoriatic skin lesions = Rheumatoid nodules - Scars 3, Palpation + Assess the temperature over the joint areas and compare these with the temperature of the forearm. + Start proximally and work towards the fingers, feeling the radial pulses and the wrist joints.’Then feel the muscle bulk in the thenar and hypothenar eminences. Ia the palms, feel for any tendon thickening and assess the sensation over the relevant areas supplied by the radial, ulnar and median nerves. + Squeeze over the row of metacarpophalangeal joints whilst watching the patient's face for any discomfort. + Bi-manually palpate MCP and interphalangeal joints. Clinical Examination 89 4, Range of Motion ‘Ask the patient to perform the following movements in the sequence mentioned below and observe for range of movement : = Make a fist — Pronate wrist — Extend little finger (extensor digiti minim is us arthritis) = Extend all fingers Assess function = Pinch grip — Opposition (touch thumb to exch finger) Power grip (ask patient to squeeze your fingers) Fromenc’s test (for ulnar nerve palsy). In this test the patient attempts to geip a paper with thumb and index finger while the examiner tries to pull the paper out of the patient's grip. — Ask patient to write something / undo a button, Assess power = Wrist extension (radial nerve) — Thumb abduction (median nerve) — Finger abduction (ulnar nerve) ly the first tendon to rupture in rheuatoid 5. Neurovascular Examination Nerve Median Ulnar Radial Sensation ‘Motor Lateral portions ofthe pulp ofthe —_| Resisted palmar abduction of the index and middle fingers thumb Lateral pulp areas of the lttlefinger | Abduction of the fingers against resistance Web space between thethumband | Wrist extension index finger (anatomical snuff box) 6. Special Tests Phalen's test : Forced flexion of the wrist, cither against the other hand or by the examiner for 60 seconds will recreate the symptoms of carpal tunnel syndrome. Finkelstein’s test is used to diagnose DeQuervain's tenosynovitis, Patient is told to flex the thumb and clench the fist over the thumb followed by ulnar deviation. If there is an increased pain in the radial styloid process and along the length of the extensor pollicis brevis and abductor pollicis longus tendons, then the test is positive for De Quervain's syndrome. “Tinel’s sign : Use the index finger to tap over the carpal tunnel at the wrist. A positive test results when the tapping. causes tingling or paresthesia in the arca of the median nerve distribution, which includes the thumb, index finger, and middle and lateral half of the ring finger. A positive Tine!’s sign at the wrist indicates carpal tunnel syndrome. 90 NAC OSCE | A Comprehensive Review Breast Examination Introduce yourself :“Iam Dr.___, your attending physician and Ill be examining you today, At any point of the examination you feel uncomfortable, please let me know and I'll stop the examination right there.” ‘Wash/Sanitize hands Explain to the patient what you are about to do and gain informed consent. Verhalize the steps of the examination and your findings. Ask which side the problem is. ‘Make sure patient is adequately exposed, use proper draping techniques 1. General Inspection (with patient sitting on side of bed) Inspect with : = Patient's arm by their sides. = Patient's arms behind their head (tenses skin) = Patient's hands on their hips (tenses pectoralis major) ‘These manoeuvers test for'T4 disease ~ invasion of chest wall / skin. Inspect for = Obvious masse: = Scars = Radiotherapy tattoos Skin changes = Peau d'orange — Dimpling = Nipple retraction — Pagers disease. 2. Inspection (with patient lying down) = Breasts size, symmetry and contous. = Arcola pigmentation, nipple pigmentation, shape, ulceration and discharge. = Skin color, thekening, venous pattern and edema. ~_ Palpate normal breast followed by abnormal breast — Palpate all quadrants, nipple and axillary tail of each breast. = Describe any masses : position, size shape, mobility, number, tenderness, consistency. — Palpate axillary, supraclavicular and infraclavicular lymph nodes, 4, Auscultate lungs, Clinical Examination s1 Thyroid Examination 1. Steps before beginning examination Introduce yourself: “Tam Dr any point of the examination you fi right there. Wash/Sanitize hands Explain to the patient what you are about to do and gain informed consent. Ensure patient is adequately exposed. Look for medical equipment/therapies Show empathy. Verbalize the steps of the examination and your findings. . your attending physician and I'l be examining you today. At 1 uncomfortable, please let me know and Ill stop the examination 2. Inspection ‘Ask for patient's vitals. ‘Observe patient : Is patient anxious? Weight guin/loss? Note hoarseness of voice. Peel pulse ~ rate/shythm/volume Face: Facial expression (dull in hypothyroidism) Periorbital myxedema Loss of 1/3rd of eyebrows Hair ~ texture/alopecia Exophthalmos (look from behind patient), lid lag Ophthalmoplegia (ask patient to follow your finger the Chemosis (cedness and watering of eyes) Hands : Temperature, moist palms, texture, color, Assess for fine tremors, palmar eythema Pemberton's sign - is the development of facial flushing, veins, inspiratory stridor and elevation of the jugular venous pressure (JVP) upon raising of the patient's both arms above his/her head simultancously. (Thoracic inlet obstruction — eg, due to goitre) Carpal Tunnel Syndrome (Tinel’s and Phalen’s Test) ~ associated with hypothyroidism, ‘Arm reflexes — brisk in hyperthyroidism. Neck: Stand in front of the patient, inspect for neck swellings/goitre ~ scars, sinuses, dikated vessels ask for diplopia) istended neck and head superficial 92 NAC OSCE | A Comprehensive Review 3. Examination of the Thyroid gland and cervical lymph nodes. + Swallow tests — Ask patient to swallow water and observe for movement of any masses. + Tongue protrusion — Thyroglossal cyst moves on tongue protrusion. + Stand behind the patient and palpate, Assess size, texture, smoothness, margins and mobility of the thyroid gland (including when swallowing), Note the temperature over gland and adjacent skin. + Palpate cervical lymph nodes. + Pereuss over sternum — Retrosternal goitte. + Auscultate for thyroid bruit ~ Grave's disease 4, Examination of legs. + Pretibial myxoedema + Peripheral edema due to congestive cardiac future. + Delayed relaxation of ankle reflex in hypothyroidism. 5. Thank the patient after the examination. Clinical Examination Mini Mental State Examination |LORIENTATION What is today’s date? Whats the year? ‘Whats the month? ‘Whats the day today? Can you tell me what season tis? Can you also tell me the name of the location we are in? (Hospital/clnic) What floor are we on? What city are we in? What country are we in? What state are we in? I, IMMEDIATE RECALL ‘Ask the patient if you may test his/her Ball memory. Say thewords “ball” , ‘flag’ , fi “tree” clearly and slowly. Then ask the s patient to repeat the words. Check for each Tree Correct response. The first repetition = determines the score. Ifthe patient does not repeat al three correctly, keep saying them up to six tries until the patient can repeat them, II ATTENTION AND CALCULATION ‘A. Counting Backwards Test Record each response Ask the patient to begin with 100 93 and count backwards by 7. Record z ‘each response. Check one box at right for each correct response. The 79 score is the number of correct Maximum score = 3 Number of Trials: _ Maximum score = 5 subtractions. 2 65 B. Spelling Backwards Test ‘Ask the patient to spell the word D 1 "WORLD" backwards. Record each _ response, Use the instructions to L 1 determine which are correct a responses, and check one box at - | a right for each correct response. ° 1 93 94 NAC OSCE | A Comprehensive Review C. Final Score Compare the scores of the Counting Backwards and Spelling Backwards tests. Write the greater of the two scores in the box labeled FINAL SCORE at right, and use tin deriving the TOTAL SCORE. IV. RECALL | Ask the patient to recall the three ‘words you previously asked him/her toremember. Check the Box at right for each correct response. V. LANGUAGE Naming Show the patient a wrist watch and ask him/her what its. Repeat for a pencil. Repetition | Ask the patient torepeat “Noifs,ands, or Correct repetition buts.” Pencil ‘Three - Stage Command the patient a sheet of blank paper and say, “Take the paper in your right/left hand, fold itn halfand put it on the floor.” Puts paper on the Reading “Hold up the card that reads, “Close your eyes.” So the patient can see it clearly. Ask himiher to read it and do what it says. Check the box at right only if he/she actually closes his/her eyes. ‘Writing Give the patient a sheet of blank paper and ask him/her to writea sentence. ttis to be written spontaneously. ifthe sentence contains a patient and a verb, and is sensible, check the box at right, Correct ‘grammar and punctuation are not necessary. Copying Show the patient the drawing of the intersecting pentagons. Ask him/her to ‘draw the pentagons (about one inch each side) on the paper provided. ten angles are present and two intersect, check the box at right. lgnore tremor and rotation. Closes eyes Writes sentence Establish the patient’s dominant hand. Give Takes paper in hand Folds paper in half Copies pentagons Final Score: _ (Max of 5 of Greater of the two scores) ‘Maximum score = 3 floor a Clinical Examination 95 DERIVING THE TOTAL SCORE ‘Add the number of correct responses, The maximux is 30. 23-30 Normal 2-19 Borderline Less than 19 Impaired Thisisa blank poge Clinical Cases Clinical Cases — Protocol for history taking 99 Protocol for history taking ‘A candidate gets 2 minutes outside the station to read the clinical case senatio on the door before entering. It is nntial to get yourself organised in these 2 minutes, + Read the question properly, understand the requirement and follow instructions (e.g. if you are asked to do a physical examination, do not start taking history. You will be losing valuable time) + You will be given a pencil and a booklet with blank pages. It is.a good practice to jot down notes. + Write the name, age, sex and chief complaint of the patient + Forhistory of preseat illness, you can use the mnemonic OCDPQRSTUV+AAA, * Past and Social History : PAM HUGS FOSS + Write down your differential diagnosis. So your note should look something like this Name of the patient, age, sex and chief complaints Q- Onset P~ Past medical history C- Course A- Allergies D- Duration ‘M - Medication P- Place H — Hospitalizations Q- Quality U- Urinary problems R™ Radiation G-GIT problems S- Severity S- Sleep T-Timing F - Family history U—You (associated with your daily activities) O- Obstetrical history V — Deja vu (Fas it happened before?) S— Social history A~ Aggravating factors S~Sesual history ‘A Allotanng fico “A~ Associated symptoms Differential diagnosis : + Knock the door before entering, relax, take + Introduce yourself to the patient: + “So, Mr/Miss deep breath, smile and enter the room with confidence. Hand over the stickers to the physician examiner, “Good morning/afternoon/evening, Mr/Mliss yTam ind I'm your attending physician today.” , what complaint bas brought you here today?" Dr___ + Show empathy : “Oh! [am sorry to hear that, I shall try my best to help you.” 100 NAC OSCE | A Comprehensive Review Data Collection : OCDPQRSTUV+AAA. + Onset: When did it start? Sudden? Gradual? + Course : Is it getting worse, better or just the same? + Duration : You said it started ..... ago, does it come and go? How often does it come? For how long does it stay each time? + Place : Show me exactly where it hurts? + Quality :Tell me how it feels like? Is it sharp/burning/dull/crampy? + Radiation : Does it travel to anywhere? + Severity : On a scale fiom 1 to 10, with 1 being the mildest and 10 the worst pain, how bad is it? + ‘Timing : Is it worse at a particular time of the day? + U (Sou) your daily activities : Docs it imerfere in your day to day activities? Does it change with your daily activities like posture, rest, eating, exertion? + V (Deja vu) : Has it happened before? When? What happened then? What medication? + Aggravating factors : What makes it worse? + Alleviating factors : What make it better? * Associated symptoms : Have you noticed anything that occurs with it? + AL: Associated constitutional symptoms like fever, shortness of breath, cough, nausea, vomiting, diarrhea, headache, fatigue (FSC NVD HF) + A2: Associated symptoms to particular system Respiratory : Chest pain, shortness of breath, cough, spurum, wheezing, runny nose, post nasal drip, contact with il person, night sweats, questions for pulmonary embolism (leg pain, long travel, surgery and OCP use in females) CVS = chest pain, orthopnea, paroxysmal nocturnal dyspnea, palpitations, tachycardia, GIT : pain in abdomen, stool, bowel movements, vomiting, jaundice, blood in stools, diet, wavel. ‘Neurology : headache, loss of consciousness, weakness, paresthesias Higher Mental Function : orientation, memory, consciousness, Motor : weakness of limbs. Sensory : tingling sensation. Cerebellum : gait, balance. Cranial Nerves : speech, swallowing, sion, hearing, PAM HUGS FOSS + “Ok, Mr/Miss ____, Now T need to ask you about your health in general. Is that okay with you?” + Past Medical History : What other medical problems do you have? (Diabetes/Hypertension/Asthma / Cancer?) + Allergies : Do you have any allergies? Are you allergic to any drugs? Clinical Cases ~ Protocol for history taking 101 + Medication : Do you take medicines at present? + Hospitalizations : (medical/surgical/trauma) + Usinary problem : burning sensation while passing urine? Blood in urine? + GIT : bowel movements? Loose stools? Constipation? Blood in stools? © (Steger + “Tam going to ask you a few personal questions that will help me in my diagnosis. Is that okay with ‘you? Let me begin by asking you about your family health.” + Family history : similar complaints in the family? Cancer in the family? Depression? Suicide? + Obstetrical History : When was your last pap smear? Wast it normal? Any history of STIs? + "Now I need to ask you about your sexual health, Whatever you tell me will be kept confidential, Is that okay with you?” + Sexial History : Are you in a physical relationship? How long? Do you practice safe sex? ‘Any tisk of STIs? + Social History : Smoking, how many packs? Alcohol, amount? Social History : Si 1g, how many ps i Sr ctyent history Route ‘Addiction/toxicity/amount Pattern of use Prior abstinence 2 Effects of the drug, D: Duration of use Recreational drugs? + “Is there anything else that you want to share?” + WRAP UP 102 NAC OSCE | A Comprehensive Review Tammy Robbins, a 48 years old lady presented with heart racing and chest discomfort for the past 3 days. Take a focused history and perform focused physical examination. Vitals: BP - 90/70 mtn Hg, HR - 146/min, irregular, RR - 12/min,’Temp - 37.5°C Clinical Info: Ms Tammy Robbins isa known hypertensive with CAD for the past 10 years, who presented with sudden onset of palpitations and chest discomfort for the past 3 days. Her symptoms are for the past 24 hours. She has dyspnea. She has dizziness for the past 12 hours. Pedal edema is 2 +. She had 2 vessel angioplasty done S year ago. 'G shows absent P waves with irregular narrow QRS complexes. Bilateral basal rales preseat on lung auscultation ical Case : Atrial Fibrillation (examination on page 65) HOPI Past History + OCD PORST UV + AAA + Do you have any medical illnesses? + How did it start? Sudden or gradual + Are you allergic to any medications? + Isit getting worse/better or no changes in + Any’surgeries in the past? the symptoms? + Past h/o recurrent int + Duration of palpitations? * Associated with chest discomfort? Family and Social Histo + Describe the type of chest discomfort? + Do you smoke? Duration & frequency. * Do you have chest pain? + Do you consume alcohol? Duration & + Any shortness of breath? frequency, + Any dizziness/light headedness? + Do you take any recreational drugs? + Any fever/cough/nausea/vomniting? + Any family history of cancers/ medical * Any hemoptysis? illnesses? Any chills night sweats? Any malaise/{atigue/weakness? + Any swelling of feet? + Any numbness/paresthesias? + Any visual problemse + Any relieving factors? + Any aggravating factors? + Any recent trauma? + List of current medications/compliance? Differential Diagnosis Management ‘Atrial brillation secondary to: aa 1. Congestive heart failure. : 2. Ischemic heart disease. + Rate control by beta blockers, calcium 3. Hypertension. channel blockers or digoxin. 4. Thyroid di Anticoagulation with heparin, then warfarin. Rhythm control by electro or medical Investigations cardioversion. +” CBC, electrolytes, glucose. | + Assess Stroke risk by using CHADS2 score. + LPT,RFT,TSH. * CK, LDH, Cardiac enzymes, + Dlead ECG Echocardiogram. + Chest X ray. Clinical Cases - Medicine 103 Simon Charles, a 20 years old male presented to your clinic with shortness of breath for the past 24 hours. “Take a focused history and perform focused physical examination. Vitals: BP - 110/80 mm Hg, HR - 110/min, RR ~ 22/min, Temp ~ 37. Clinical Info: Mr Simon Charles has a h/o of Asthma since the past 10 years, He recently cleaned his basement 1 day ago and his asthma symptoms exacerbated. He is having wheezing, chest tightness,cough and SOB leis currently on inhalers with no night symptoms. On cxumination, he has dyspnea and wheezing present in all lung fields, He has mild exacerbation of his symptoms and needs only outpatient treatment. Clinical Case : Asthma (examination on page 69) HOPI Past History + OCD PORST UV +AAA + Do you have asthma/other allergies? + How did it start? Sudden or gradual + Are you on any medications and compliance + Isitgeeting worse/better or no changes in + Are you allesgic to any medications? the symptoms? + Any hospitalizations for asthma? + Do,you wakeup in night with shortness of + H/oany other medical illness? cath? + Do you have noisy breathing? | Family and Social Histor + Hfo fever/sore throat/rash? + Do you sobke? Datixina Aegan + Any night time cough? + Do you consume aleahol? Duration & + Any sputum production? frequency. + Any chest pain with deep inspiration? + Do you use reereational drugs? TRAPPED. + Any recent activity which worsened your + Any family history of asthma/allorgies? symptoms? + Any exposure to cold sir/dust/mites? + Any pets at home? + Any changes in bowel & urinary habits? * Any contact with a sick person? © Any recent travel? + Any recurrent episodes? + Do symptoms affect your daily activities? Differential Diagnosis Management o Asthma | ee entolin 2-4 puffs MDI gd-6h + Acute bronchitis. + Pluticasone 2-4 puffs bid Inhaled steroids) * GERD. * Asthma education: + Pneumonia, Inhaler use. Spacer and holding chamber use. Investigations Symptom monitoring. + CBC. Early recognition of exacerbation. + Pulmonary function tests, Avoid environmental allergens. + Peak flow meter. Encase matizess and pillow in + Chest X ray, impermeable cover. Reduce indoor humidity to <50%. 104 NAC OSCE | A Comprehensive Review Larry Edwards, a.55 years old man presented with blood in sputum and shortness of breath for the past 5 days; Take a Focused histony and perfor focus! physical = 96/min, RR - 18/min, Vitals: BP - 160/110 mm Hg, H1 amination ‘emp ~ 37.5°C. Clinical Info: Mr Larry Edwards is a known hypertensive vio pmeenied with shortness of breath and blood in sputum for 5 days. It is gradual in onset. He has chest pain his malta, He io Losartan, non compliant wid nia. He is. No fever or recurrent pneur s not taken pirin, Atorvas, multi vitamins. He h his anti-hypertonsives for 4 weeks. Has paroxysmal nocturnal dyspnea and orthopnea. hyp paroxys lysp! et Clinical HOPI + OCD PORSTUV+AAA + How did it start? Sudden or gradual. + Isit getting worse/hetter or no changes in the symptoms? Duration of blood in sputum? Amount / color of blood of sputum? Any of shortness of breath? Present at rest or with exertion? Wo orthopnea? Vo paroxysmal nocturnal dyspnea? Any Lier pata Shima " ‘Type of pula ~ duatp or dul? Docs the pain radiate or shoot anywhere? Severity of pain on a scale of 1-10. ‘Do your symptoms change with time?? H/o fever/cough/sore throat/rash? Any chills/night sweats? “Any swelling of face or feet? Any change in weight/appetite? Any bruises on the body? ‘Any trauma recent Differential Diagnosis + Pulmonary Ede: * Pulmonary embolism. + Bronchiectasis. Drug induced coagulopathy. Pneumonia Investigations +" CBC, clectrolytes, glucose. LFT, RFT. PT,PTT INR. 12 lead Et Echocardiogram, Chest X Ray. : Congestive Heart Failure (examination on page 65) cory ‘Do you have diabetes or hypertension? Are you on any medications? Compliance with medications? y ic to any medications? + Any surgeries in the past? + Past hv recurrent iniections? Family and Social Histo + Do you smoke? Duration & frequency. + Do you consume alcohol? Duration & frequency. + Do you use recreational drugs? TRAPPED. ‘Any family history of cancers/ medical illnesses ‘Any family history of heart disease? Management +" Symptomatic treatment. + -dmit in cardiac care unit. Inj Lasix 40 mg 1V stat Beta blockers & ACE inhibitors Tnj Morphine 2-4 mg LV. Nasal oxygen. + Sublingual Nitroglycerines, + Position ~ 45 degree clevation of head end. ical Cases - Medicine 105 Jack Allen, 2 65 years old man presented with sudden onsct of right arm weakness 4 hours ago. Take a focused history and perform focused physical examination, Vitale BP 160/90 mn Lig, HR = 96/min, RR - 12/ein,Temp ~ 37.5°C. 1: Mr Jack Allen presented with sudden onset of right arm weakness with numbness and hours ago. He fn slurring of specch, blurring of vision and mild headache. He has no womiting or head trauma. No weakness of lower limbs or Je ast 10 years and non compliant to medications. Clinical Case : Cerebrovascular Attack (examination on page 71) arm. No incontinence. He is hypertensive HOPI Past History + OCD PORSTUV +AAA + Do you have diabetes or hypertension? + Tow did it start? Sudden or gradual. + Areyou on any medications/compliance? + Isit getting worse/better or no changes in * Are you allergic to any medications? the symptoms? + Any surgeries in the past? + Duration of weakness? + Where is the weakness located? Familyand Socal History + Is there any associated pain in the arm? + Do you smoke? Duration & frequency. + Any numbness/tingling/paresthesias? + Do you consume alcohol? Duration & + Any limitations in right arm movements? frequency. + Any abnormal position of right arm? + Do you use recreational drugs? TRAPPED. + Do your symptoms change with time? + Any family history of cancers! medical = Any changes in vision? illnesses? + Any changes in speech? + Any dficy alo ing/drooling of saliva? + Any he: + Any weakness of left arm or lower limbs? * Any seizures? + Any changes in facial expression? + H/o fever/cough/sore throat/rash? * Any chest pain/shortness of breat + Any changes in bowel & urinary habits? Differential Diagnosis Management + Cerebrovascular Attack. + Admit urgently. + Transient Ischaemic Attack. + Urgent neurological consult. + Sub Arachnoid Hemorrhage. + Neurovirals q 1 hourly. + Nasal oxygen. Investigations + Tab Aspirin 325 mg PO OD. + CBC, ESR, Glucose, + Blood pressure control. + Lipid profile, RFT: + Thrombolysis to be done only if presented + PepPrINr. within 3 hours of onset of symptoms. + CT's MRI Head. + Echo, Carotid doppler. 106 NAC OSCE | A Comprehensive Review ‘Allan Smith, a 70 years old man presented with light headedness and di history and perform focused physical examination. ness for 2 days.’Take a focused Vitals: BP - 110/80 mm Hg, HR - 56/min, irregular, RR - 12/min, Temp ~ 37.0°C. Clinical Inf experiencing light headedness and dizziness weeks. He i changes or 1G shows Type 2 Second Degree AV block. inical Case : Digoxin Toxicity (examination on page 65) breathless on cxertion, Has 2+ pedal e Mr Allan Smith is a known hypertensive for the past 20 years on medications. He is Te the past 2 days. He has palpitations and mild chest pain for 2 fama, Ne fainting episode. No trauma recently. No visual mb weakness. He is on Losartan, Ramipril, Digoxin, Atorvas, Aspirin, Nexium and Calcium. HOPI Past History + OCD PORST UV + AAA + Do you have diabetes or h + How did it star? Sudden or gradual + Are you allergic to any m + Isitgetting worse/better or no changes in + Any surgeries in the past? the symptoms! + Past h/o recurrent infections? Duration of dizziness? Any relieving factors Any aggravating factors? ‘Any episode of fainting? : Any weakness/tngling paresis of Hs? ‘Any chest pain/palpitations? : ‘Any shortness of breath? . ‘Any visual changes/headache? i loss/ear discharge? ‘Any excessive sweating? Any fever/cough/vomiting? ‘Any abdominal pan/oss of apes” ‘Any changes in bowel/urinary habits? Any swelling of ankles? ‘Do your symptoms vary with change in head osition? + List of current medications & compliance? ‘Any changes in medications & dosages? ‘Any rece wana? | Differential + Digoxin toxicity. + Arhythmia, : + TIA : + Ansty disorder. + Medication induced bradyeardia. . Investigations + Serum Digoxin level, CBC, electrolytes, RFT. \R/PTT, glucose. \G, 24 hour Holter monitor. Echocardiogram,Carotid Doppler. Family and Social Hist + “Do you smoke? Ds ruration & frequency. Do you consume alcohol? Duration & frequency. Do you use recreational drugs? TRAPPED. Any family history of caneers/ medical illnesses? in cardiac care unit. Stop Digoxin. Stare Digibind in case of massive overdose or refractory toxicity External Pacemaker. Clinical Cases - Medicine 107 Lisa Giroux, a 25 years old lady presented with lump in the neck for the past 7 days.’Take a focused history and perform focused physical examination. Vitals: BP - 120/88 mm Hg, HR — 96/min, RR ~ 12/min, Temp ~38.5°C. Clinical Info: Ms Lisa Giroux noticed 2 lumps on the right side of her neck below the mandible. She has positive history of fever for 5 days,sore throat and fatigue. On examination she has 2 enlarged, tender submandibular mph nodes, Monodpot test spose Clinical Case : Infectious Mononucleosis (Sore throat ) Hopi + OCD PQRST UV + AAA + How did't start? Sudden or gradual. + [sit getting worse/better or no changes in the symptoms? Since how long have you noticed the lumps? Where are the lumps located? How does the lumps feel like? Is there any pain associated with the lumps? ‘Type of pain - sharp or dll pain? Does the pain radiate or shoot anywhere? Severity of pain on a scale of 1-10. Do your symptoms change with time? H/o fever/cough/sore throat/rash? Any chest pain/shortness of breath? ‘Any abdominal pain? Any weakness of myalgia? Any chills/night swe: Any change in voice? Any change in appetite? ‘Any change in weight? Any changes in bowel & urinary ha Any contact with a sick person? Recent travel? Differential Diagnosis 2 TafacH a ieaiuelaoa + Acute bacterial pharyngitis. + Acute viral pharyngitis. + Lymphoma. + Toxoplasmosis. Investigations > CBC, + Monospot test. + Throat swab for gram stain, culture & sensitivity. Past History + Do you have diabetes or hypertension? + Are you on any medication: + Are you allergic to any medications? + Any surgeries in the past? + Past h/o recurrent iniectio Famlyand Social History, + Do you smoke? Duration & frequency: + Do you consume alcohol? Duration & frequency. + Do you use recreational drugs? TRAPPED, + Any family history of cancers/ medical illnesses? Management + Symptomatic treatment. + Maintain adequate hydration. + Rest. + Analgesics for pain + Treat Streptococcal pharyngitis, if coexists + Awoid active sports, + Steroids used only for severe symptoms. 108 NAC OSCE | A Comprehensive Review Jason Hardinge, a 26 years old university student wants focused history and address his concerns. Vitals: BP - 120/88 mm Hy, HR - 88/min, RR - 12/mi Clinical Info: Mr Jason Hardinge is having difficul months. He is currently in a monogamous relationship in maintaining to discuss confidential issues with a doctor. Take jin, Temp - 37.5°C i etetion during intercourse forthe past 4 with his girlfriend. He is on Paroxetin for his mood. disorder for 6 months. No other medical illnesses. Girlfriend is very understanding. He has no morning or night cumescence. Has no erection with self stimulation, He is very anxious about this issue. Clinical Case : Impotence HOPI + OCD PORST UV + AAA How did it start? Sudden or gradual Is it getting worse/better or no changes in the symptoms? Duration of impotence? ‘Course & frequency of impotence? Description of the problem: no erection at all, cannot sustain erection, ejaculate too quickly to satisfy partner? ‘annot achieve orgasm or orgasm without ejaculation? ny retrograde ejaculation? Circumstances under which impotence occurs only with certain partners, only at certain times or locations, what percentage of | the time? Is impotence related to lack of sexual desire? Presence and firmness of morning or nocturnal erections Can sustain erections with self stimulation? Associated problems: anxiety attacks, anhedlonta! epresuon, periued or pahtal numbness, poor peripheral circulation. Diagnosis Impotence secondary to antidepressants, Causes of impotence : (IMPOTENCE) Tatogenie, Mecham, Paychologeal, Occlusive vascular, Trauma, Extra factors, Neurogenic, Chemical, Endocrine, Investigations +” CBC, blood glucose, TSH Se Testosterone. Urinalysis Endocrine lab tests, if indicated! FSH,LH, Prolactin. Sexual orientation ~ heterosexual/homoscxual/bisexual? Current relationship? Performance anxiety? Any stresses in the relationship? Any stresses at home or scho Sexual partner's expectations? ‘Are you on any medication Compliance/type of medications ~ de tory Do you have diabetes or hypertension? Are you allergic to any medications? “Any surgeries in the past? ‘Any trauma in the past? Past h/o recurrent infections? Past Hist Family and Social Histor + Do you smoke? Duration 8: frequency. Do you consume alcohol? Duration & frequency. Do you use secreational drugs? TRAPPED. Any family history of cancers/medical illnesses? Management +” Complete physical exam, Reassurance, counseling both patient & artner. Fform that symptoms are due to side effects of anti-depressants. Symptoms are reversible by changing the doscor the type of drug. Substitute with another anti-depressant: Minimal to no sexual dysfunction Nefazodone (Serzone) Bupropion (Wellbutrin) Low risk of sexual dysfunction (10-15%) Fluvoxamine (I.uvox) Citalopram (Celexa) Venlafaxine (Effexor) Avoid alcohol/smoking. Medical weatment: Tab Sildenafl 25-50 mg PO 0.5 to 4 hours prior to coitus. 109 Clinical Cases - Medicine - Taylor Jackson, a 18 years old boy presented with fever, neck stiffness and photophobia to the ER. "Take a facused history and perform focused physical examination. Vitals: BP - 90/70 min Hyg, HR - 110/min, RR ~ 12/min, Temp ~39.0°C. Clinical Info: Wet for the past 1 day. He is alert "Taylor Jackson has high grade fever for the pas < conscious. No setaures. H 3 days along with neck stiffness. He has is Gsalosn ythcmaced A lo ear discharge. Has a purpuric rash on chest and lower limbs, No recent trauma, Has h/o contact with ak oO; with similar sympto: Ehinical Case: Meningitis HOPI + OCD PORSTUV+ AAA + Tow did fever start? Sudden or gradual + Isitgetting worse/berteror ne changes the symptoms? + Duration of fever? Continnous/intermittent/remittent? Duration of neck stiffness Associated with headache/nausea/vomiting? Any photophobia/phonophobia? ‘Any iguelmalaise? ‘Any fainting/scizures/confusion/irritability? H/o fever/cough/sore throat/rash? Any chills/night sweats? Any chest pain/shortness of breath? Any abdominal pain? Any changes in bowel & urinary habits? Any contact with a sick person? Recent trav Recent head trauma? Differential agnosis acterial Meningitis + Encephalitis. + Intra-cerebral abscess. lnvestigations CBC, electrolytes, glucose. + RFT,ABG. + Lumbar puncture - gram stain, microscopy, culture/sensitivity (rule out papilledema). + Blood culture/sensitivity, + Urinalysis. CT scan head. Chest X Ray Febrile, Brudzinski's and Kernig’s sign are positive. tion on page 71) Past History ‘Do you have medical illnesses? ‘Are you on any medications? Are you allergic to any medications? Any at geil epee Past ho recurrent infe Family and Social Histo Deyou smoker Dorion fedieauency Do you consume alcohol? Duration & Frequency. Do you tise recreational drugs? TRAPPED. Any family history of cancers/ medical illnesses? ‘Management ‘Admit under isolation. + Start empiric antibiotics prios to LP. Taj Cefotaxime 2 ¢ IV qh + Inj Ampicillin 50 mg/kg IV qoh, Inj Dexamethasone 10 mg q6h IV x4 days, ‘Treat all close contacts. 110 NAC OSCE | A Comprehensive Review James Irwin a 30 years old man presented to your clinic with symptoms of headache, Take a focused history and address his concerns. Vitals: BP - 120/88 mm Hy, HR - 96/min, RR - 12/min,’Temp = 37.5°C. Clinical Info: Mr James Irwin presented with unilateral, pulsating headache, grace 7/10 for the past 6 months, He experiences aura prior to the onset of headache. Associated with nausea,vomiting and photophobia, Stimulated by stress and excessive caffeine intake. One episode lasts for 8-12 hours. He had 6 attacks in past 6 months, Currently on Advil prn. Clinical Case : Headache (Migraine) HOPI + OCD PORSTUV+AAA + How did it start? Sudden or gradual. * Location of pain? + Isit getting worse/better or no changes in the symptoms? | + Since how long have you noticed the pain? + Number of episodes of headache till now? + How many hours pain lasts? + How does the pain feel like? + Does the pain radiate or shoot anywhere? + Severity of pain on a scale of 1-10. + Any provoking factors like food, alcohol, caffeine, chocolate, wine, ste: + How does the pain decrease’ + Doyour symptoms change with time? | + Ho fever/cough/sore throat/nasal stuffiness? + Any excessive lacrimation? Any visual changes prior to onset of heddachee een Any neck pain/rigidity/seizures? + Any weakaess/muscle pain/paresthesias? + Any mood changes/decreased sleep/energy? | Differential Diagnosis + Migraine with Aura + Tension Headache. + Cluster Headache. ‘Temporal Arteritis Investigations + CBC,ESR Past History Do you have diabetes or hypertension? ‘Any head trauma in the past? ‘Are you on any medications? Axe you allergic to any medications? Any surgeries in the past? Pur h/o recument iabstions? Family and Social Hist +” Do you smoke a Duis & frequency. + Do you consume alcohol? Duration & frequency. + Do you use recreational drugs? TRAPPED. ‘Any fannly history of cancers/ medical illnesses? ‘Management +” Symptomatic treatment. + Avoid triggers. + Mild attack - ASA, Ibuprofen. + Moderate attack ~ Anti-emetics, Ibuprofen, Triptans, ergots. + Severe attack - Anti-emetics, Exgotamines, ‘Triptans. + Migraine prophylaxis ~ Propranolol, Amitriptyline, Verapamil, Clinical Cases - Medicine 11 Michael Smith, a 55 years old man presented with chest discomfort for the past 1 hour. Take a focused history and perform focused physicll examainution, Vitals: BP - 160/90 mm Hg, HR - 96/:in, RR ~ 12/min, Temp ~ 37.5°C. Clinical Info: Mr Michael Smith presented with left sided chest discomfort for the past 1 hour. He has pain jelt shoulder and jaw. He has shortness of breach along with palpitations. He is a known hypertensive and diabetics on oral medications. His wife states he is non-compliant with his medications. ECG shows ST elevation in leads II IIL and avF. Clinical Case : Chest Pain (Myocardial Infarction) (examination on page 65) HoPI Past History * OCD PORST UV + AAA ‘Do you have diabetes or hypertension? + Hovr did it start? Sudden or gradual. + Are you on any medications/compliance? + [sit getting worse/better or no changes in + Are you allergic to any medications? the symptoms? + Any surgeries in the past? * Duration of chest discomfort? * Past h/o recurrent infections? * Location of chest discomfort? * — How does the discomfort feel like? Family and Social Histor 3) (Any pain associitesd with ches dicciitante 7 Dowie Dacwonsereeace | + Does the pain radiate or shoot anywhere? + De youconsumealeahol? Duron & + Severity of pain ona scale of 1-10. frequency. + Any shortness of breath? + Do you use recreational drugs? TRAPPED. + Any palpitations/diaphoresis/dizziness? + Any family history of cancers/ medical + Da your symptoms become change with illnesses? foe + H/o fev ee throat/rash? © Any hem = Suerte bua |= Any abdominal pain? | + ‘Any weakness of myalaia? + ‘Anychaniges in bowel 8 urinary hal + Any swelling of feet? + Any relieving Bictor | + Any aggravating factors? Differential Diagnosis Management ‘Myocardial Infarction. 2° Admit torcardniccareunit: + Unstable Angina. + Urgent cardiology consult, + Pericarditis. + Nasal oxygen + Panic Attack. + Tab Aspirin 160-325 mg chewable stat. + Inj Morphine 2-4 mg IV stat 8 pra Teas + Sublingual Nitroglycerin 0.4 mg stat, repeat CBC, clecenilyreapllvcase, sim ® + INR/PTT. * Inj Atenolol 5 mg IV bolus over 5 mins, then + Serial CK“MB & Troponin q8h X 3. repeat. + ABG. Then switch to oral Atenolol 50 mg. + CXR + Canliae monitoring every 30 minutes-1 * | 12 lead ECG. hour. + Bed rest. * Consider PCI, in case of no contraindications. 112 NAC OSCE | A Comprehensive Review Adam Sawyer, 18 years old male presented with fever, cough for 1 week along with shortness of breath. Take a focused history and perform focused physical examination. Vitals: BP - 116/70 mm Hg, HR - 96/min, RR - 20/min, Temp ~ 38.5°C. Clinical Info: Adam Sawyer has fever and expectorant cough for the past 1 week. Hee has whe shortness of breath for 2 days. On auscultation of chest, there is decreased bre: rales present Clinical Case : Pneumonia (examination on page 69) h sounds on left side with HOPL ory | + OCD PQRST UV + AAA Do you have diabetes or hypertension? + How did it start? Sudden or gradual. * Are you on any medications? + [sit getting worse/better or no changes in ‘Ate you allergic to any medications? | the symptoms? ‘Any surgeries in the past? + Duration of cough? + Past h/o recurrent infections? + ‘Type of cough - dry or expectorant? + Isthere any chest pain? Family and Socal History + Location of chest pain? +” Do you smoke? Duration & frequency. + ‘Type of pain ~ sharp or dull? + Do you consume alcohol? Duration & + Does the pain radiate or shoot anywhere? frequency. + Severity of pain on a scale of 1-10. + Do you use reereational drugs? TRAPPED. + Pain present with deep breathing® + Any family history of cancers/ medical + Onset of shortness of breath? esses? + Any noisy breathing present? + Any chills/night sweats * Any changes in bowel & urinary habits? © Any contact with a sick person? + Recent travel? | Differential Diagnosis Management + Community Acquired Pneumonia. +" Outpatient treatment. + Acute exacerbation of COPD. + Tab Doxycycline 100 mg PO bid x 7 days. + Acute bronchitis. + ‘Tab Azithromycin 500 ng PO OD x 5 days. + Asthma + Rest and adequate hydration. + Symptomatic treatment. Investigations + Tapatient treatment for 48-72 hours. + CBC. : Levofloxacin 750 mg PO q24h. + Sputum gram stain with culture/sensitivity. ‘Tab Amoxicillin 1000 mg PO gid + + Hlectraites + Tab Clarithromycin 500 mg PO bid. * Renal function tests. * Chest X Ray. Clinical Cases - Medicine 113 Helen Solazzo is an ICU nurse who had a ncvdle stick injury 30 minutes ago while drawing blood sumple from a patient . Take a focused history and address her concerns, Vitals: BP - 120/88 mm Hg, HR - 86/min, RR - 12/min, ‘Temp ~ 37.0°C. Clinic ago ing blood had suse ek injury in the TCU 30 nat me, She was wearing gloves, She has no high risk behavior. Her immune status for HIWAICV/IIBsAy is negative as of 1 year ayo. Patients immune status is unknown as of now. Physician examiner Clinical Case Diagnosis: Post exposure prophylaxis for ITV. HOPI Past History + Mechanism of injury. + Do you have medical illnesses? + Was the nurse wearing gloves? + Are'you on any medications? + Were adequate occupational precautions: + Ace you allergic to any medications? taken? (like gloves, mask, + Any surgeries in the past? + Depth of needle penetration? + Pasth/o recurrent infections? + Type of needle- hollow or solid? + Needle gauge? Pamily and Social Hist oo ee ee * Any contact with patient's body fluids? * Do you consume alcohol? Duration & + Steps taken after injury? frequency. + Immune status of the nurse? + Do you use recreational drugs? TRAPPED. + When was the last immune status checked? + Any family history of eancers/ medical + Immunization for Hepatitis A and Hepatitis illnesses? BP . pay igh risk behavior like unprotected sex? : ;milar incident in the past? . Q the immune status of the patient known? + Patient's medical history? | Investigations Management For Healthcare profesional +7” Reassurance, nlytes. e + HIV,HCV, HBsAg, For the patient: inform the pt. . + CBC, electrolytes + HIV, HCV, HBsAg. = + Incase of positive HIIV/HICW/FIBsAg do viral loads & CD 4 counts. : that the patient's result come back positive for HIV, Refer to Infectious Disease clinic. Report to occupational health dept within 72 hours Scevery 2 weekly, core to worker's compensation board for file claim. Advise about safe sex practices, Avoid pregnaney/breast feeding. Repeat blood work 6 weeks,12wecks,6 months and 12 months. Patient HIV + then start the nurse on post exposure prophylaxis for 4 weeks. Counsel about side effects of medications, 14 NAC OSCE | A Comprehensive Review Jacob Sandler, a 50 years old man presented with hemoptysis and right sided calf swelling for the past 2 days. He had knee replacement surgery 1 week ago, Take a focused history and perform focused examination, Vitals: BP 140/80 mm Hg, TTR - 110/min, RR ~ 18/min, Temp —37.5°C. Clinical Info: Mr Jacob Sandler had a right knee replacement 1 week ago. He now presented with 2 episodes of hemoptysis and right calf swelling with tendemess. He has no fever or infection of surgical wound, Homan’s sign is positive with ECG showing S1Q3T3 pattern. Clinical Case : Pulmonary Embolism (examination on page 69) (HOPI Past History + OCD PQRSTUV + AAA + Do you have diabetes or hypertension? + How did it start? Sudden or gradual. + Are you on any medications? + Isit getting worse/better or no changes in + Are you allergic to any medications? the symptoms? + Any surgeries in the past? Durition of blood in sputum? + Past h/o recurrent infections? + Amount / color of blood of sputum? + Onset of shortness of breath/chest pain? Family and Social Hist + Present at rest or with exertion? + “Do you smoke? Duration 8 frequency. + H/oorthopnea? + Do you consume alcohol? Duration & + H/o paroxysmal nocturnal dyspnea? frequency. + Duration of leg swelling? + Doyou use recreational drugs? TRAPPED. + Where is the leg swelling located? + Any family history of cancers/ medical * Any leg pain associated with swelling? illnesses? + ‘Type of pain - sharp or dull pain’ Docs the pain radiate or shoot anywhere? Severity of pain on a scale of 1-10. Any pain during rest? Any pain in the night time? ‘Any skin discoloration of legs/nail changes? Any skin ulceration of legs? Any fever/cough/cold: Any headache/dizzin Any weakness/muscle pain Any prolonged immobit ‘Any pain/discharge from the wound? ‘Any pre-op or intra-op complications? Differential Diagnosis Management * Balonary embilism, + Admit in Intensive care unit. + Deep Veit Thrombosis. + Elevate head endl 10 45 degree. + Pulmonary edema due to CHE + Nasal oxyg + Myocardial infarction, Give chewable ASA 160-325 mg immediately. Secure IV access, bolus IV Lasix 40 mg, CBC, RFT. Ventolin if wheezes are heard, Electrolytes, glucose. Sublingual nitro spray, if blood pressure is Serial CK-MB and Troponin qh X 3. adequate. Arterial blood gc Tnj Morphine 1 mg LV. PT.PTT,INR, factor assay. Investigations D. + Inj Heparin 7500 U IV bolus, then infuse at hocardiogram.. 1200 U/h, then switch to warfarin, Chest X Ray. + Continue anticoagulation for 3 months. PET Scan or VQ scan Doppler of lower limbs. + ECG, Clinical Cases - Medicine 5 Jasper Preudhomme, a 16 years old boy is a known epileptic presented to your clinic for the first time. Take a focused history and address his concerns. Vitals: BP - 120/88 mm Hg, HR - 96/min, RR - 12/min, Temp ~ 37.0°C. Clinical Info: Mr Jasper Preudhomme is a known epileptic for the past 6 years. He is on regular anti- epilepties and ompliant. His last seizure was 2 months ago. He recently started consuming alcohol with friends. His main concer is to get a driver's license. Clinical Case : Seizure disorder HOPI Past History + Age of onset + Do you have diabetes or hypertension? + Precipitants: Sleep deprivation, drugs, + Any surgeries in the past? alcohol, T'V screen emotional upset. + Past h/o recurrent infections? + Describe type of seizures. + Salivation, cyanosis, tongue biting, Family and Social History incontinence, automatisms, motor ¥8. +” Do you smoke? Duration & frequency. visual/gustatory/olfactor * Do you consume alcohol? Duration & + Frequency & duration of seizures. frequency. + What body parts affected and in what order? | + Do you iise recreational drugs? TRAPPED. + Premonitory signs (presence of aura: implies | + Any family history of cancers/ medical foeal atari. illnesses? + Post-ictal state (decrease in level of consciousness, heudache, sensory phenomens tongue soreness pains ‘odds paralysis ~ hemiplegia) + Degree of control achieved with \edications. + Wasa CT scan done when seizures were first diagnosed? | + Number and deseription of recent seizures. + Are they different from previous seizures? + Isthe patient having any sew symptoms like headache, vomiting, new newologteal deficits? + Side effects of antiepileptics: drowsiness, Jor concentration, poor performance in COS ipeckaiss pee pera esr ae, nystagmus, dysarthria, hypertrichosis, gingival hypertrophy Diagnosis Management er «Disease sf medians + Regular follow up. Investigations + Avoid alcohol consumption/smoking, + "CBC, electrolytes. + Avoid recreational drugs. + Serum deug levels + Inform to the patient Ministry of + EEG. ‘Transportation regulations requize patient to be seizure free for I year or more. + Notify Ministry of Transportation as required by law. 116 NAC OSCE | A Comprehensive Review Samantha Tlo, a 56 years old woman presented to your clinic with symptoms of headache and blurry vision ‘Take a focused history and address her concems Vitals: BP - 130/88 mm Hg, FIR - 86/min, RR ~ 12/min,Temp ~ 37.5°C. Clinical Info: Ms Samantha Ho presented with unilateral, left temporal side pulsating headache, grade 7/10 for the past 2 weeks, She experiences headache while chewing and combing her hait: Associated with blurring of vision and diplopia. One episode lasts for 30 minutes. Currently on advil pra, atenolol 50 mg OD and multivitamins. Clinical Case : Temporal Arteritis HOPI Past listory * OCD PORST UY + AAA Do you have diabetes or hypertension? + How did it start? Sudden or gradual | Any head trauma in the past? * Location of pain? * Arc you on any medications? + [sit getting worse/better or no changes in + Are you allergic to any medications? the symptoms? + Any surgeries in the past? + Since how long have you noticed the pain? + Past h/o recurrent infections? + Number of episodes of headache till now? + For how long the pain lasts? Family and Social History, + How does the pain fec! like? +” Do you smoke? Duration & frequency. + Does the pain radiate or shoot anywhere? + Do you consume alcohol? Duration & + Severity of pain on a scale of 1-10. frequency. + Any provoking fictors ike fond kohol, Do you use recreational drugs? TRAPPED. | caffeine, chocolate, stress, combing hair. + Any family history of cancers/ medical | + How does the pain decrease? illnesses? | * Do your symptoms change with time? + H/o fever/cough/sore throat/nasal stuffiness? + Any excessive lacrimation? | + Any visual changes with headache? | + “Any neck pain/sigidity/seiares? + Any weakness/muscle pain/paresthesias? + Any mood changes/decreased sleep/enengy? Differential Diagnosis ‘Management + Temporal arteritis. + High dose corticosteroids, + Migraine. + Tab Prednisone 60 mg PO OD until + TL symptoms subside and ESR is norma, then 40 mg PO OD for 4-6 weeks. Then taper to Investigations 5-10 mg PO OD for 2 years. + CBC, ESR, CRP, LF + Relapses occur in 50% iF treatment is + ‘Temporal artery biop: stopped before 2 years. + Visual acuity. + Monitor ESR regularly. + Fandoscopy. + TF visual symptoms present, then admit &. start Inj Prednisolone 1000 mg IV qi2h for 5da

You might also like